102
F-ULL-LENGTH SIMULATED TOEIC TESTS Simulated TOEIC Test 1 Simulated TOEIC Test 2

practical guide to the new toeic_practice tests.pdf

Embed Size (px)

Citation preview

Page 1: practical guide to the new toeic_practice tests.pdf

F-ULL-LENGTH

SIMULATED TOEIC TESTS

Simulated TOEIC Test 1

Simulated TOEIC Test 2

Page 2: practical guide to the new toeic_practice tests.pdf
Page 3: practical guide to the new toeic_practice tests.pdf

TEST 1 Questions

Part 1: Photographs

Part 2: Question-Response

Part 3: Short Conversations

Part 4: Short Talks

Part 5: Incomplete Sentences

Part 6: Text Completion

Part 7: Reading Comprehension

Page 4: practical guide to the new toeic_practice tests.pdf
Page 5: practical guide to the new toeic_practice tests.pdf

~ 3 = ;' ~ Q.

.-3 0 t"'j ..... t"':l

z 0 i= u w Ill (!) z z w t;; :::::i

z 0

t w Ill (!) z c <( w a:

Part 1

No. ANSWER

ABCD

1 ®®©@ 2 ®®©@ 3 ®®©@ 4 ®®©@ 5 ®®©@ 6 ®®©@ 7 ®®©@ 8 ®®©@ 9 ®®©@ 10 ®®©@

Part 5

ANSWER No.

ABCD 101 ®®©@ 102 ®®©@ 103 ®®©@ 104 ®®©@ 105 ®®©@ 106 ®®©@ 107 ®®©@ 108 ®®©@ 109 ®®©@ 110 ®®©@

Part 2

No. ANSWER

ABC No.

ANSWER

ABC

11 ®®© 21 ®®© 12 ®®© 22 ®®© 13 ®®© 23 ®®© 14 ®®© 24 ®®© 15 ®®© 25 ®®© 16 ®®© 26 ®®© 17 ®®© 27 ®®© 18 ®®© 28 ®®© 19 ®®© 29 ®®© 20 ®®© 30 ®®©

No. ANSWER

ABC D No.

ANSWER

ABCD

111 ®®©@ 121 ®®©@ 112 ®®©@ 122 ®®©@ 113 ®®©@ 123 ®®©@ 114 ®®©@ 124 ®®©@ 115 ®®©@ 125 ®®©@ 116 ®®©@ 126 ®®©@ 117 ®®©@ 127 ®®©@ 118 ®®©@ 128 ®®©@ 119 ®®©@ 129 ®®©@ 120 ®®©@ 130 ®®©@

TOEIC® Simulated Test

ANSWER SHEET

Part 3

ANSWER ANSWER ANSWER ANSWER No. No. No. No.

ABC ABCD ABCD ABCD

31 ®®© 41 ®®©@ 51 ®®©@ 61 ®®©@ 32 ®®© 42 ®®©@ 52 ®®©@ 62 ®®©@ 33 ®®© 43 ®®©@ 53 ®®©@ 63 ®®©@ 34 ®®© 44 ®®©@ 54 ®®©@ 64 ®®©@ 35 ®®© 45 ®®©@ 55 ®®©@ 65 ®®©@ 36 ®®© 46 ®®©@ 56 ®®©@ 66 ®®©@ 37 ®®© 47 ®®©@ 57 ®®©@ 67 ®®©@ 38 ®®© 48 ®®©@ 58 ®®©@ 68 ®®©@ 39 ®®© 49 ®®©@ 59 ®®©@ 69 ®®©@ 40 ®®© so ®®©@ 60 ®®©@ 70 ®®©@

Part 6 Part 7

No. ANSWER

ABC D No.

ANSWER

ABCD No.

ANSWER

ABCD No.

ANSWER

ABC D

131 ®®©@ 141 ®®©@ 151 ®®©@ 161 ®®©@ 132 ®®©@ 142 ®®©@ 152 (A) (B) (c) (D) 162 ®®©@ 133 ®®©@ 143 ®®©@ 153 ®®©@ 163 ®®©@ 134 ®®©@ 144 ®®©@ 154 ®®©@ 164 ®®©@ 135 ®®©@ 145 ®®©@ 155 ®®©@ 165 ®®©@ 136 ®®©@ 146 ®®©@ 156 ®®©@ 166 ®®©@ 137 ®®©@ 147 ®®©@ 157 ®®©@ 167 ®®©@ 138 ®®©@ 148 ®®©@ 158 ®®©@ 168 ®®©@ 139 ®®©@ 149 ®®©@ 159 ®®©@ 169 ®®©@ 140 ®®©@ 150 I®®©© 160 ®®©@ 170 ®®©@

io-l ~ TOEIC" IS a registered trademark of the Educational Testmg Service (ETS) This publication IS not affiliated with. endorsed. or approved by ETS. ~ ....

(..)

Part 4

No. ANSWER

ABCD No.

ANSWER ANSWER No.

ABC D ABCD

71 ®®©@ 81 ®®©@ 91 ®®©@ 72 ®®©@ 82 ®®©@ 92 ®®©@ 73 ®®©@ 83 ®®©@ 93 ®®©@ 74 ®®©@ 84 ®®©@ 94 ®®©@ 75 ®®©@ 85 ®®©@ 95 ®®©@ 76 ®®©@ 86 ®®©@ 96 ®®©@ 77 ®®©@ 87 ®®©@ 97 ®®©@ 78 ®®©@ 88 ®®©@ 98 ®®©@ 79 ®®©@ 89 ®®©@ 99 ®®©@ 80 ®®©@ 90 ®®©@ 100 ®®©@

ANSWER ANSWER ANSWER No. No. No.

ABCD ABC D ABCD

171 ®®©@ 181 ®®©@ 191 ®®©@ 172 ®®©@ 182 ®®©@ 192 ®®©@ 173 ®®©@ 183 ®®©@ 193 ®®©@ 174 ®®©@ 184 ®®©@ 194 ®®©@ 175 ®®©@ 185 ®®©@ 195 ®®©@ 176 ®®©@ 186 ®®©@ 196 ®®©@ 177 ®®©@ 187 ®®©@ 197 ®®©@ 178 ®®©@ 188 ®®©@ 198 ®®©@ 179 ®®©@ 189 ®®©@ 199 ®®©@ 180 ®®©@ 190 ®®©@ 200 ®®©@

Page 6: practical guide to the new toeic_practice tests.pdf

58 Practical Guide to the NEW TOEIC® Test

Page 7: practical guide to the new toeic_practice tests.pdf

LISTENING TEST

In the Listening test, you will be asked to demonstrate how well you understand spoken English. The entire Listening test will last approximately 45 minutes. There are four parts, and directions are given for each part. You must mark your answers on the separate answer sheet. Do not write your answers in your test book.

PART 1

Directions: For each question in this part, you will hear four statements about a picture in your test book. When you hear the statements, you must select the one statement that best describes what you see in the picture. Then find the number of the question on your answer sheet and mark your answer. The statements will not be printed in your test book and will be spoken only one time.

Sample Answer

Example

Statement (B), "They're performing at a nightclub," is the best description of the picture, so you should select answer (B) and mark it on your answer sheet.

GO ON TO THE NEXT PAGE

Simulated TOEIC Test 1 Listening 359

2

3

4

5

6

7

Page 8: practical guide to the new toeic_practice tests.pdf

1.

2.

360 Practical Guide to the NEW TOEIC® Test

Page 9: practical guide to the new toeic_practice tests.pdf

3.

;« en .... -

~ IS!

sw 2 ~;s

3

4

5

6

7

4.

Simulated TOEIC Test 1 Listening 361

Page 10: practical guide to the new toeic_practice tests.pdf

5.

6.

362 Practical Guide to the NEW TOEIC® Test

Page 11: practical guide to the new toeic_practice tests.pdf

7.

8.

Simulated TOEIC Test 1 Listening 363

-2

3

4

5

6

7

Page 12: practical guide to the new toeic_practice tests.pdf

9.

10.

364 Practical Guide to the NEW TOEIC® Test

Page 13: practical guide to the new toeic_practice tests.pdf

PART2

Directions: You will hear a question or statement and three responses spoken in English.

They will not be printed in your test book and will be spoken only one time. Select the best

response to the question or statement and mark the letter (A), (B), or (C) on your answer

sheet.

Example

You will hear: When does the next train arrive?

You will also hear: (A) In about five minutes.

(B) You can get there by bus.

(C) It's not far from here.

Sample Answer

e ®©

The best response to the question "When does the next train arrive?" is choice (A), "In

about five minutes," so (A) is the correct answer. You should mark answer (A) on your

answer sheet.

11. Mark your answer on your answer sheet. 26. Mark your answer on your answer sheet.

12. Mark your answer on your answer sheet. 27. Mark your answer on your answer sheet.

13. Mark your answer on your answer sheet. 28. Mark your answer on your answer sheet.

14. Mark your answer on your answer sheet. 29. Mark your answer on your answer sheet.

15. Mark your answer on your answer sheet. 30. Mark your answer on your answer sheet.

16. Mark your answer on your answer sheet. 31. Mark your answer on your answer sheet.

17. Mark your answer on your answer sheet. 32. Mark your answer on your answer sheet.

18. Mark your answer on your answer sheet. 33. Mark your answer on your answer sheet.

19. Mark your answer on your answer sheet. 34. Mark your answer on your answer sheet.

20. Mark your answer on your answer sheet. 35. Mark your answer on your answer sheet.

21. Mark your answer on your answer sheet. 36. Mark your answer on your answer sheet.

22. Mark your answer on your answer sheet. 37. Mark your answer on your answer sheet.

23. Mark your answer on your answer sheet. 38. Mark your answer on your answer sheet.

24. Mark your answer on your answer sheet. 39. Mark your answer on your answer sheet.

25. Mark your answer on your answer sheet. 40. Mark your answer on your answer sheet.

GO ON TO THE NEXT PAGE

Simulated TOEIC Test 1 Listening 365

1

3

4

5

6

7

Page 14: practical guide to the new toeic_practice tests.pdf

PART3

Directions: You will hear some conversations between two people. You will be asked to answer three questions about what the speakers say in each conversation . Select the best response to each question and mark the letter (A}, (B), (C), or (D) on your answer sheet. The conversations will not be printed in your test book and will be spoken only one time.

41. What was the article about? 44. Where are the speakers?

(A) A coupon for sale items (A) At a festival (B) Their competitor's sale (B) In a school (C) A story about Mr. Roberts (C) At a council meeting (D) Summer vacation (D) In a library

42. Why is the man shocked? 45. What kind of books does the man need?

(A) They received new merchandise. (A) Books on aviation

(B) Mr. Roberts will have a sale. (B) The Wright Brothers' life story

(C) Their competitor is not closing. (C) Festival and celebration books

(D) The ad was in the newspaper. (D) City council history

43. What will the two speakers do next? 46. Where can the man find the books he

(A) Meet with their boss needs?

(B) Put an ad in the newspaper (A) On the seventh floor (C) Have a meeting (B) On the second floor (D) Inform Mr. Roberts immediately (C) On the third floor

(D) On the fifth floor

366 Practical Guide to the NEW TOEIC® Test

Page 15: practical guide to the new toeic_practice tests.pdf

47. Why did the man reschedule his 53. Where does the conversation take appointment? place?

(A) He got an emergency call. (A) At the man's office (B) Mrs. Wilson was busy. (B) At a box company (C) He had to use the back door. (C) In an elevator (D) He'll work out at the gym. (D) In a hospital

48. Why did Mrs. Wilson ask the man to 54. Why did the man go to the doctor? come over? (A) He popped his knee. (A) To thank him in person (B) He would move some boxes. (B) To have her pipes checked (C) He had back pain. (C) To repair her stairs (D) He broke his leg. (D) To replace her windows

55. What seems to be the reason for the 49. When will the man stop by Mrs. Wilson's man's injury?

place? (A) Carrying lots of boxes (A) After the emergency (B) Moving to a new house 4 (B) Five minutes later (C) Working in an office

5 (C) Tomorrow afternoon (D) Checking his reflexes (D) This afternoon 6

7 56. What are the speakers discussing?

so. Why did the man want to meet the (A) Starting their own business woman? (B) Solving financial problems

(A) There's a problem with the (C) Hiring short-term employees

bathrooms. (D) Getting a raise

(B) The woman paints houses for a 57. How was the budget changed? living.

(C) The man needs a new office. (A) It includes special fees.

(D) The man is apartment hunting. (B) It was totally taken away. (C) It increased by $20,000.

51. What has recently been done to the (D) It was cut down by $20,000. space?

58. (A) The walls have been painted.

What will the speakers probably do next?

(B) The walls have been separated. (A) They'll meet the company heads.

(C) A private office has been built. (B) They'll start over again.

(D) Bathrooms have been added. (C) They'll find new jobs. (D) They'll lay off more people.

52. What information is the man most interested in?

(A) If the bathrooms are separate (B) The office's location (C) How much the rent is (D) The size of the main area

I GO ON TO THE NEXT PAGE

Simulated TOEIC Test 1 Listening 367

Page 16: practical guide to the new toeic_practice tests.pdf

59. How will the man get to his destination? 65. What is the man probably in charge of? (A) By train (A) Accounting (B) By boat (B) Personnel management (C) By plane (C) Hotel maintenance and repairs (D) By car (D) Banquet reservations and planning

60. Where is the man most likely heading? 66. What was the woman's problem?

(A) To a remote island (A) She had unexpected guests. (B) She needed to cancel her (B) To a big city

appointment. (C) To the countryside

(C) She made the reservation too late. (D) To a boat manufacturing factory (D) She added too many tables.

61. Who is Kiel Miller? 67. How did the man solve the problem?

(A) The man's new customer (A) He spoke to his manager. (B) A boat captain at the dock (B) He returned the call as soon as (C) The woman's husband possible. (D) The head of the new office (C) He made the reservation a week

earlier. (D) He reserved three extra tables for

62. What is the purpose of this conversation? the woman.

(A) To say hello to Dr. Richman (B) To contact the man (C) To arrange travel plans 68. Who most likely are the speakers?

(D) To reschedule an appointment (A) Husband and wife

63. When is the man's new appointment? (B) Teacher and student (C) Boss and employee

(A) Monday morning (D) Athlete and coach (B) 4:00 this afternoon (C) Tomorrow at 9:00a.m. 69. Who is waiting for the fax? (D) 9:00 this evening (A) The man

64. What kind of appointment does the man (B) The company next door (C) Ms. Boil

have? (D) Mr. Stevens

(A) A physician appointment (B) A dentist appointment 70. Where will the fax probably get sent (C) An eye appointment from?

(D) A loan appointment (A) The office next door {B) The speakers' office (C) Washington (D) The woman's home

368 Practical Guide to the NEW TOEIC® Test

Page 17: practical guide to the new toeic_practice tests.pdf

PART4

Directions: You will hear some talks given by a single speaker. You will be asked to answer three questions about what the speaker says in each talk. Select the best response to each question and mark the letter (A), (B), (C), or (D) on your answer sheet. The talks will not be printed in your test book and will be spoken only one time.

71. What is discussed in the manual? 74. Where is this announcement most likely

(A) Piloting an airplane being made?

(B) The workshop schedule (A) On an ocean cruise (C) Repairing emergency lights (B) On a tour bus (D) Emergency procedures (C) During a flight

(D) During a lunch party 72. What is shown on page three?

(A) Different models of airplane 75. What will the people take their picture

(B) Two sequences of pictures with?

(C) Procedures for fire emergencies (A) A famous kind of plant (D) Water landing instructions (B) A rocky highway

(C) The tour guide 73. Who would be listening to this talk? (D) A cluster of people

(A) First-time pilots (B) Flight attendants in training 76. What are the people reminded to do?

(C) Retired boat captains (A) Venture out alone (D) Newly hired travel agents (B) Leave their cameras behind

(C) Stay in pairs or groups (D) Collect pretty rocks

I GO ON TO THE NEXT PAGE

Simulated TOEIC Test 1 Listening 369 '

2

3

5

6

7

Page 18: practical guide to the new toeic_practice tests.pdf

77. What are the listeners asked to do? 83. What kind of announcement is this?

(A) Clean the carpets by hand (A) A traffic advisory (B) Clean up their workspaces (B) A severe weather update (C) Put papers into piles (C) A travel promotion (D) Give Gary their files (D) An emergency alert

78. What will be done tonight? 84. What will begin on May 16th?

(A) The carpet will be cleaned. (A) Shuttle buses will start running . (B) The district manager will come. (B) Interstate 90 will be reopened. (C) Papers will be recycled . (C) Sections of the highway will be

(D) Chairs will be replaced . closed. (D) The detours will end.

79. What is Gary probably in charge of? 85. Where can residents get more (A) District management information? (B) Office supplies

(A) From local newspapers (C) Cleaning the carpet

(B) On state radio programs (D) Taking out the trash

(C) From the City Planning Office (D) At the construction sites

80. What is being advertised?

(A) New morning fitness classes 86. Where would this announcement be (B) Coupons for the Garden Restaurant heard? (C) A discount on full-package stays

(A) At the zoo (D) Remodeled hot spring pools

(B) On an airplane

81. Which class is offered in the morning? (C) In a sports stadium (D) At a music concert

(A) Cooking (B) Meditation 87. When would this announcement most (C) Massage likely be made? (D) Swimming (A) At the end of the championship game

82. How long is the package being offered? (B) During the winners' ceremony (C) After the Grand Prize was given

(A) For 165 days away

(B) All year round (D) Before the championship game

(C) This weekend only started

(D) This month only 88. How could listeners get free snack coupons?

(A) By going to the gift shop (B) By purchasing other snacks first (C) By signing up for them (D) By buying a T-shirt or a hat

370 Practical Guide to the NEW TOEIC® Test

Page 19: practical guide to the new toeic_practice tests.pdf

89. How long has the speaker worked in the 95. What is the purpose of this talk?

company? (A) To get building plans under way

(A) Twenty years (B) To seek help for the fundraiser

(B) One year (C) To inform and thank volunteers

(C) Ten years (D) To choose a Partners in Care

(D) Two years sponsor

90. Who is Mr. Trump? 96. How did the Children's Center get money c;t to expand?

(/1

(A) The speaker's assistant ..... ~

(B) The speaker's boss (A) Their sponsor endorsed the plan. (C) The speaker's father (B) They held a fundraiser. (D) The event host (C) They applied for a loan. 1

(D) The committee members approved 2 91. What is the main purpose of this it.

speech? 3

(A) To announce retirement 97. Where will the new center be located?

(B) To accept an award (A) On the west side of town 5

(C) To introduce a guest speaker (B) On the east side of town

(D) To congratulate a performer (C) In the downtown area 6

(D) In the neighboring city 7

92. Where is this short talk most likely being made? 98. According to the report, what recently

(A) On the radio happened to James Smith?

(B) At a private meeting (A) He received millions of dollars.

(C) In the newspaper (B) He was arrested by police.

(D) At a school (C) He designed a computer program. (D) He became company president.

93. What will the Lincoln Bakery do?

(A) Close for remodeling 99. For how long had Mr. Smith been taking

(B) Move to a different location money?

(C) Close its store for good (A) 2 years (D) Open a new branch (B) 12 years

(C) 5 years 94. What will open in the historical society? (D) 8 years

(A) The town's new bakery (B) An exhibit about the Lincoln Bakery 1 00. What was a result of the news?

(C) A recipe exchange party (A) James Smith was put in jail. (D) The annual baking competition (B) Millions of dollars were stolen.

(C) Company shares lost value. (D) Mr. Smith was promoted.

This is the end of the Listening test. Turn to Part 5 in your test book.

Simulated TOEIC Test 1 Listening 371

Page 20: practical guide to the new toeic_practice tests.pdf

READING TEST

In the Reading test, you will read a variety of texts and answer several different types of reading comprehension questions. The entire Reading test will last 75 minutes. There are three parts, and directions are given for each part. You are encouraged to answer as many questions as possible within the time allowed.

You must mark your answers on the separate answer sheet. Do not write your answers in your test book.

PARTS

Directions: A word or phrase is missing in each of the sentences below. Four answer choices are given below each sentence. Select the best answer to complete the sentence. Then mark the letter (A) , (B) , (C), or (D) on your answer sheet.

101. Unless we can be -------for an event 103. Paul Laird , the ------- of the two new like this and change our strategies, the employees, is to be promoted to chief of

company will still be in the red for the the sales section because of his superb

foreseeable future. performance on the job.

(A) prepare (A) well experienced

(B) prepared (B) most experienced

(C) preparatory (C) more experienced

(D) preparation (D) greatly experiencing

102. The economy as a whole has been 104. The general manager is planning to

declining in that country for years, ------- fly to Paris and stay there for -------

the computer industry has expanded to supervise the operation of the new

steadily. branch office.

(A) so (A) sometime

(B) yet (B) some time

(C) since (C) sometimes

(D) unless (D) some times

372 Practical Guide tq the NEW TOEIC® Test

Page 21: practical guide to the new toeic_practice tests.pdf

105. No sooner ------- New York than Ms. 109. ------- its vast population, China is now O'Dowd scheduled a meeting with Mr. the world's largest beer market by Carter on the likelihood of a merger volume, according to Euromonitor data. between the two companies. (A) Despite (A) she reached (B) For all (B) did she reached (C) Thanks to (C) had she reached (D) In order that

~ (D) was she reached 110. It is widely understood that advertising

tn -106. A peninsula surrounded by more than -------not only to promote sales but also ~

200 islands, Hong Kong is one of the to upgrade a company's image. most densely ------- urban regions on (A) helps the planet. (B) to help 2 (A) population (C) helping (B) popular (D) help 3

(C) popularity 4 (D) populated 111. To facilitate our efficiency, management

has purchased more than 50 desktop 107. The World Bank is composed of five computers, many of-------- are to be 6

branches, each ------- its own budget installed in the accounting section. 7 and carrying out separate duties. (A) them

(A) having (B) it (B) has (C) what (C) to have (D) which (D) is having

112. MPG, a diversified industrial company, 108. We regret ------- you that we do not plans to build a polysilicon plant in

accept credit card purchases for our Indonesia to ------- rising demand for line of products; however, we do accept solar energy. bank drafts drawn on a recognized (A) cope bank. (B) deal (A) to inform (C) meet (B) informing (D) content (C) to have informed (D) having informed

Simulated TOEIC Test 1 Reading 373

Page 22: practical guide to the new toeic_practice tests.pdf

113. DVDs are most effective ------- the discs 117. ------- oil prices continuing to rise, are sturdy and the crisp images do not economists predict that consumers will degrade with repeated showings. cut back on spending in other areas.

(A) in that (A) About (B) except that (B) With (C) for that (C) By (D) with that (D) In

114. The CEO------- us to work at our 118. Not satisfied with the proposal the own pace, so we can leave the office salesman had presented, the manager anytime we want to during the day. made him ------- a new one immediately.

(A) lets (A) write (B) allows (B) to write (C) makes (C) writing (D) has (D) written

115. To increase efficiency, the president 119. Since information technology makes demanded that we ------- the operating ------- increasingly easy to find and system and update most of the software work with outside resources, many we use. companies are getting smaller in size.

(A) change (A) itself (B) can change (B) it (C) changed (C) them

(D) have changed (D) themselves

116. Bear in mind that no part of this book 120. The employee was ------- excited about may be reproduced------- written his new job that he vowed to work

permission from the author. even harder to live up to the boss's

(A) unless expectations.

(B) when (A) quite

(C) without (B) very

(D) for (C) extremely (D) so

37;4 Practical Guide to the NEW TOEIC® Test

Page 23: practical guide to the new toeic_practice tests.pdf

121. Management has passed a rule 125. One of the major problems most small prohibiting employees ------- in all businesses encounter today is -------indoor places for the sake of everyone's advertising is often costly and often health. takes some time to begin working .

(A) from smoking (A) because (B) to smoke (B) that (C) by smoking (C) what

~ (D) to not smoke (D) how en ,..,. 122. As far as marketing is -------, every 126. An analyst for Apple Inc. said a way to

...;a,.

opportunity we have to meet new follow up the iPhone with a cheaper people is an opportunity to grow our version would be to ------- the Nano into business. a phone priced at $300 or lower.

2 (A) intended (A) convert (B) concerned (B) converse 3

(C) reached (C) conserve 4 (D) designed (D) convey

123. Without sufficient information, it can be 127. Mr. Wilson was quite upset upon 6 very difficult to decide ------- a particular learning that the sales section was

7 marketing program is profitable to your ------- ever present for the staff meetings business or not. on Mondays.

(A) how (A) hardly (B) that (B) nearly (C) whether (C) almost (D) where (D) practically

124. Becoming a solution provider ------- 128. The personnel manager will brief the solving a problem even if the solution new employees ------- the company's doesn't involve the purchase of your current situation when they assemble in product. the meeting room tomorrow morning.

(A) means (A) by (B) means to (B) for (C) is meant to (C) with (D) means by (D) on

I GO ON TO THE NEXT PAGE

Simulated TOEIC Test 1 Reading 375

Page 24: practical guide to the new toeic_practice tests.pdf

129. It is true that each advertising medium 133. ------- you have any problems with the is very useful to us, but each has machine, consult Mr. Paul N. Hiatt, advantages and disadvantages, ------- who is one of the most experienced it? mechanics in our factory.

(A) isn't (A) Had (B) doesn't (B) Would (C) won't (C) Should (D) haven't (D) Were

130. An increase in global temperatures 134. Almost one third of all Americans could cause many changes, among are employed in marketing-related ------- the amount and pattern of positions, so the number of possible precipitation resulting in floods and marketing careers is -------. drought. (A) enormous (A) that (B) tiny

(B) this (C) insignificant

(C) them (D) invisible

(D) which 135. Though Mr. Johnson is competent,

131. To help improve the image of the -------remains to be seen whether or not

company, the secretary ------- to write he can break the sales record for the

a brief report whenever she receives a month of August.

complaint from a customer. (A) that

(A) requires (B) what

(B) req~iring (C) it

(C) is required (D) which

(D) required 136. To tell you the truth , never before -------

132. At a press conference in New York, seen so many candidates competing for

President and CEO Robert Densaki so few vacancies our company has to

unveiled a sixteen-month project offer.

------- saving his company from going (A) we have bankrupt. (B) we had

(A) so as to (C) have we

(B) in order to (D) had we

(C) with a view to

(D) so that

376 Practical Guide to the NEW TOEIC® Test

Page 25: practical guide to the new toeic_practice tests.pdf

137. As a result of poor management, Sableline, a company once famous the world over for its quality toys, is ------- to close down in two years.

(A) likely (B) possible (C) probable (D) maybe

138. The handbook is not an easy book to read , but one ------- is very useful to anyone who wants to understand the inner workings of this company.

(A) that (B) who (C) it (D) how

139. Any proposal made by a staff member should ------- be turned down if it fails to meet the CEO's expectations.

(A) recently (B) fortunately (C) smoothly (D) definitely

140. All the letters of application should be mailed to the personnel manager -------Friday June 6 so that interviews for qualified applicants can be arranged.

(A) no more than (B) no less than (C) no later than (D) no sooner than

Simulated TOEIC Test 1 Reading 377

n;t en ..... ..II.

2

3

4

6

7

Page 26: practical guide to the new toeic_practice tests.pdf

PART&

Directions: Read the texts that follow. A word or phrase is missing in some of the sentences. Four answer choices are given below each of the sentences. Select the best answer to complete the text. Then mark the letter (A), (B), (C), or (D) on your answer sheet.

Questions 141-143 refer to the following report.

Changing the World with Chips

By Tim McDonald

According to analysts, the most significant difference between the chips that ------- today's computers and the computer chips of the future is, in a word , size. 141. (A) block

(B) power

(C) manufacture

(D) shrink

In the future, there will be hundreds of billions of embedded chips and sensing devices integrated into everything -------from key chains and swimming pools to your apartment's

142. (A) including

(B) regarding

(C) ranging

(D) covering

walls and even your skin . All of these devices will be able to compute, sense and

communicate with each other.

If that sounds creepy, consider one possible application: DARPA, the US government agency -------the Internet, is funding research into "smart dust," millions of cheap, wireless sensors 143. (A) that created

(B) created

(C) creates

(D) to create

based on microelectromechanical systems (MEMS) technology.

378 Practical Guide to the NEW TOEIC® Test

Page 27: practical guide to the new toeic_practice tests.pdf

Questions 144-146 refer to the following article.

When customers aren't happy with your business, they usually won't complain to you. -------, they'll probably complain to just about everyone else they know- and take their 144. (A) Besides

(B) Therefore (C) Instead (D) In addition

business to your competition next time. That's why an increasing number of businesses are making follow-up calls or mailing satisfaction questionnaires after the sale is made. They find that if they promptly follow up and resolve a customer's complaint, the customer might be even more -------to do business than the average customer who didn't have a complaint.

145. (A) willing (B) reluctant (C) probable (D) interested

In many business situations, the customer will have many more interactions after the sale with technical, service, or customer support people than he or she did with the sales people. So if you're serious about ------- customers or getting referrals,

146. (A) detaining (B) retaining (C) remaining (D) sustaining

these interactions are the ones that are really going to matter.

(Source: BusinessTown.com)

I GO ON TO THE NEXT PAGE

Simulated TOEIC Test 1 Reading 379

2

3

4

5

7

Page 28: practical guide to the new toeic_practice tests.pdf

Questions 147-149 refer to the following e-mail message.

To: All restaurant personnel <[email protected]> From: Bill Monroe <[email protected]> Subject: Sick Leave

Due to the large number of staff taking sick leave over the past few months, the Blue Moon Restaurant management has been forced to ------- a new sick leave policy.

147. (A) complement

(B) implement (C) supplement

(D) experiment

Staff employed for over one year are allowed five paid sick leave days per year. That will not change. -------, with the new policy, only those sick leave absences accompanied by a letter

148. (A) Though (B) Consequently (C) However

(D) Moreover

or prescription from a physician, clinic, or hospital will be ------- paid sick leave. Previously,

149. (A) rejected (B) denied (C) invited (D) granted

calling in sick would automatically qualify for paid sick leave even without medical authentication.

New staff employed for less than a year will be allowed two days of paid sick leave per year,

again , with medical authentication.

The new policy will begin on March 1.

380 Practical Guide to the NEW TOEIC® Test

Page 29: practical guide to the new toeic_practice tests.pdf

Questions 150-152 refer to the following letter.

Ms. Ruth Benedict

439 West Branch Road Yonkers, New York 1 0702

October 1

Dear Ms. Ruth Benedict:

We are pleased to inform you that you are one of the finalists in our search for someone to fill

the position of chief purser for Northeast Airlines. After--- reviewing your resume, we would 150. (A) taking care

(B) with care (C) carefully (D) caring about

like to invite you to our head office in New York for an interview.

The interview is scheduled------- Tuesday morning, October 22 at 10:00 at Suite 12-A in the

151. (A) for (B) on

(C) in

(D) at

Pre~cott Building, 927 Third Avenue in Manhattan. We have reserved a room for you at the New yorker Hotel for Monday and Tuesday evenings, October 21 and 22. Your reservation code number is A424BQ. A voucher for US$1 00 may be applied towards any meals in their hotel restaurants. Transportation to and from New York is your own responsibility.

Succe.s.sful candidates will be -------on Monday, November 3. I wish you the best of luck.

Sincerely,

G~W~ Gloria Woods

152. (A) notified

(B) reported

(C) presented (D) prosecuted

I GO ON TO THE NEXT PAGE

Simulated TOEIC Test 1 Reading 381

2

3

4

5 -7

Page 30: practical guide to the new toeic_practice tests.pdf

PART7

Directions: In this part you will read a selection of texts, such as magazine and newspaper articles, letters, and advertisements. Each text is followed by several questions. Select the best answer for each question and mark the letter {A) , (B), (C) , or (D) on your answer sheet.

Questions 153-154 refer to the following advertisement.

Sales dropping? Employees leaving?

Customers screaming?

You need the Five Secrets of Small Business!

To get the advantage over your competitors, you have to know something they don't. That's why you're invited to the Five Secrets of Small Business! This very special conference is the work of Harvard MBA Chris Reynolds, author ofbest-sellers like Business Riches. Come learn five simple ways to become rich and successful. Call 905-454-9853 for more information.

153. What is being advertised?

(A) A best-selling book (B) Harvard Business School (C) A business seminar (D) A competitor's secret information

154. What credentials are given for Chris Reynolds?

(A) His success with small businesses (B) His knowledge of company secrets (C) His position at a major university (D) His education and a book he

published

382 Practical Guide to the NEW TOEIC® Test

"

•.

Page 31: practical guide to the new toeic_practice tests.pdf

Questions 155-157 refer to the following ticket.

r------------------------------------------------------------• Northland High School 20-Year Reunion

DRINK VOUCHER I

I I

·----------------------- -------------------------------------4 Ticket Number: 612

The holder of this ticket is entitled to three free drinks during the course

of the evening. Please present this ticket to the bartender when ordering

your drink and have it validated. At the end of the evening, place the

bottom half of your ticket in the glass bowl at the counter to enter a

drawing to win an all-expenses-paid trip to the Bahamas. Please hang on

to the top portion of your ticket to claim your prize at the end of the reunion. Have fun, and we'll see you all at the next reunion.

·---------·---------·- - -------·------------------------------~ Name: __________________ ___

Cocktail Cocktail Cocktail Date:

Coupon Coupon Coupon Telephone Number: _____ _

Ticket N umber: 612

155. How many years has it been since graduation?

(A) Three (B) Twenty (C) 612 (D)1997

156. What will the ticket holder receive?

(A) Another ticket (B) A trip to the Bahamas (C) A glass bowl (D) Three free drinks

157. When will the drawing be held?

(A) In 20 years (B) At the end of the month (C) When the evening comes to an end (D) At the next reunion

Simulated TOEIC Test 1 Reading 383

1

2

3

4

5

6 -

Page 32: practical guide to the new toeic_practice tests.pdf

Questions 158-160 refer to the following e-mail message.

Myron Applebaum, Euro Travel

Sally Legazia, IMT International

Itinerary for European Trip

March 14, 9:45.25 a.m.

Dear Ms. Legazia:

Attached please find a copy of your itinerary. I was able to make all the reservations for

your one-week trip in England, Germany, and Austria except for two. (1) The flights

on your preferred carriers from London to Berlin were all full for Monday morning,

March 27, both economy and business class. I can still get you on a low-cost carrier for

that morning, or, if you prefer, I can get you on an afternoon flight at 1 :45 from London

to Berlin on SAS. Please let me know immediately. (2) Your requested hotel in Vienna

does not have any suite vacancies for March 31-April 1. The Edelweiss Hotel, a four­

star hotel in the suburbs, does. I can get you a large double room in your preferred

downtown hotel. I would advise the latter because transportation on your tight schedule

from suburbs to city could be a problem.

As soon as I hear from you, I will make the changes suggested above or try any new

ones you bring forward. When you are thoroughly satisfied, I will send you the bill for

airlines and hotel reservations. Thanks for thinking of us at EuroTravel.

384 Practical Guide to the NEW TOEIC® Test

Page 33: practical guide to the new toeic_practice tests.pdf

158. What is the purpose of the e-mail?

(A) To invite a friend on a trip (B) To request travel decisions (C) To demand payment for services (D) To suggest first-class seating

159. In the e-mail , what does the term "SAS" in paragraph 1, line 6, refer to?

(A) An airline carrier (B) A type of airline seating (C) The name of a country (D) The name of an airport

160. What is the problem with hotel

accommodations in Vienna?

(A) The suites available are too

expensive. (B) The double room available is too

expensive. (C) No suite is available in the

requested hotel. (D) No suite is available in suburban

hotels.

I GO ON TO THE NEXT PAGE

Simulated TOEIC Test 1 Reading 385

1

2

3

4

5

6

Page 34: practical guide to the new toeic_practice tests.pdf

Questions 161-165 refer to the following survey results.

Entertainment World Yearly Recap

100

80

60

40

20 0

-Less than $10,000

1st Qtr

2nd Qtr

.---

3rd Qtr

~

--------$10,000-

$25,000

$25,001-

$50,000

Monthly Income

386 Practical Guide to the NEW TOEIC® Test

4th Qtr

~ -.. -More than

$50,000

• Movies

• Plays

D Rentals

-+--Movies ... Plays __.__ Rentals

Page 35: practical guide to the new toeic_practice tests.pdf

161. Which quarter saw the greatest 164. What happens to the rate of video

difference between forms of rentals as income increases?

entertainment viewed? (A) It increases.

(A) The 3rd quarter (B) It remains the same.

(B) The last quarter (C) It decreases.

(C) The 2nd quarter (D) It changes both ways.

(D) The 1st quarter 165. How is the group that makes more than ;t

162. When were film rentals at their lowest? $50,000 a month different from the (/) I"+

others? """" (A) In the last quarter (B) In the 3rd quarter (A) They go to the movies more often

(C) In the 2nd quarter than they rent movies. 1

(D) In the beginning of the year (B) They watch more plays than movies. 2

(C) They attend plays less often than

163. Which income group attends the they watch movies. 3

greatest number of plays? (D) They rent movies fewer times than 4 (A) The lowest income group they go to plays.

5 (B) The highest income group (C) People making between $25,001 6

and $50,000 per month (D) People making less than $10,000

per month

GO ON TO THE NEXT PAGE

Simulated TOEIC Test 1 Reading 387

Page 36: practical guide to the new toeic_practice tests.pdf

Questions 166-168 refer to the following memo.

MEMORANDUM

TO: All Section B-4 Employees ,-FROM: Mary Hightower RE: Relocation of Office Space

DATE: Wednesday, July 6

We are going to relocate to new office space before the end of this month in order to start business in the new space on Monday, August 1. I know this is relatively short notice, but I know you'll be delighted with the larger workspaces as well as the better view!

The new office space for B-4 is on the 11th floor of the Parker Building on

Renault Road, just a few minutes from here. The new location offers better parking opportunities. It seems likely that everyone will get a personal parking space!

The movers are coming on July 28 and 29. Until then, pack only non-essential items. Packing materials will be made available from Monday, July 11. A meeting to discuss any moving problems will be held this Friday at 2:00p.m.

See you then.

Mary Hightower

166. What is the purpose of the memorandum?

(A) To notify employees of a move (B) To inform employees of a business

opportunity (C) To request a moving company to

move an office (D) To direct employees to new parking

spaces

167. When will the move be finished?

(A) July 6 (B) July 11 (C) July 29 (D) August 1

168. Who most likely is Mary Hightower?

(A) A Section B-4 secretary (B) A moving company employee (C) A real estate agent (D) A company manager

388 Practical Guide to the NEW TOEIC® Test

Page 37: practical guide to the new toeic_practice tests.pdf

Questions 169-170 refer to the following billing statement.

BOWERS AND SONS, CRAFTSMAN TOOLS

2306 Capitol Road Columbus, Ohio 43226

(614) 337-3123 (614) 337-4488 (fax) www.bowersandsons.com

Invoice# 1549 Date: February 15 To: Mr. Arnold Buckley

217 Radcliffe Lane Lancaster, Ohio 43130

Item

Electric hand-held drill

Concrete bolts (2")

8-foot ladder

Electric hand-held sander

Utility hammer

Coltrane Tool kit

Billing Statement

Quantity Unit Cost

1 $49.99

1 (package) $9.99

1 $17.99

1 $29.99

1 $13.99

1 $45.99

Total

Applicable State Tax

Total

$49.99

$9.99

$17.99

$29.99

$13.99

$45.99

$168.94

$10.48

Grand Total

Visa

$179.42

- $179.42

Payment Due $000.00

Note: Returned personal checks are subject to a 5% penalty. All major credit cards accepted. See us about our installment plan for purchases of up to $500.

169. What was the most expensive item Mr. Buckley purchased?

(A) An electric hand-held drill (B) An electric hand-held sander (C) An eight-foot ladder (D) A tool kit

170. How did Mr. Buckley purchase his goods?

(A) In cash (B) By personal check (C) By credit card (D) By installments

Simulated TOEIC Test 1 Reading 389

2

3

4

5

6 -

Page 38: practical guide to the new toeic_practice tests.pdf

Questions 171-173 refer to the following notice.

Important Information for Use of Bacitracin Gel

The tube of Bacitracin Gel you are holding is the finest product of its kind on the

market. As with all medicines, keep this tube out of the reach of children. If taken

orally, call your physician, your local poison control center, or 911 and follow their

instructions faithfully.

The active ingredient in Bacitracin Gel is sensitive to heat. Keep this tube out of

direct sunlight or in temperatures above 30°C (88°F) for prolonged periods.

This is a topical application; it is not for oral use under any circumstance. Its

primary utility is to protect against infection in minor cuts, scrapes, and burns (1st

degree). For more serious injuries, consult your local physician, a health clinic or

hospital, or call 911.

We guarantee the safety and utility of this product when used as directed. Forward

any complaint to Masters Pharmaceuticals, 711 Devon Road, Dover, Delaware

19901. For a replacement or refund, include tube with a description of your

complaint and the purchase receipt. For further information on the product or on

filing a complaint, contact us at our toll-free number 1-800-MASTERS or online at

www.masterspharm.com.

390 Practical Guide to the NEW TOEIC® Test

Page 39: practical guide to the new toeic_practice tests.pdf

171. What is the main purpose of the notice?

(A) To explain how bacitracin works (B) To advise consumers on product

use (C) To protect the company against

legal action (D) To tell consumers how to get a free

product

172. What should be done if a child accidentally swallows the product?

(A) Contact emergency services (B) Keep away from heat (C) Avoid direct sunlight (D) Call a toll-free number

173. What are dissatisfied customers of Bacitracin Gel advised to do?

(A) Call their local health clinic or doctor (B) Sue Masters Pharmaceuticals in

court (C) Take the product orally at once (D) Return the product for a refund

Simulated TOEIC Test 1 Reading 391

2

3

4

5

6 -

Page 40: practical guide to the new toeic_practice tests.pdf

Questions 174-177 refer to the following minutes of a meeting.

Supervisors Quarterly Meeting

October 25

08:32- VP L. Hamilton begins the meeting and notes· that six people are present.

08:35 - L. Iiamilton gives quarterly performance briefing by department.

H u man Resources: The number of employee complaints this quarter was

very low. Employee satisfaction events have been very successful.

Accounting: Both company and customer paperwork was completed very

slowly this quarter. The new Q-Form system does not work very well.

Sales: Sales were down this quarter. Many new customers opened contracts

with the company; however, several larger old accounts were lost.

D esign: Several new employees were hired this quarter and they have

produced very good work. However, the rate of production needs to

increase.

08:53 - The meeting is opened for discussion. Sales Supervisor K. Janes suggests

that Sales design a program of benefits for loyal customers.

08:57 - Accounting Supervisor I. Falis says that Accounting has a program for

keeping track of customer satisfaction and offers assistance.

09:00- K. Janes thanks I. Falis for the offer and suggests th~t they meet to disC!J.SS

it.

09:01 - L. Hamilton asks if there is any more discussion,

09:02 - L. Hamilton adjourns the meeting.

392 Practical Guide to the NEW TOEIC® Test

Page 41: practical guide to the new toeic_practice tests.pdf

174. How long did the meeting last?

(A) An hour (B) A quarter of an hour (C) Three quarters of an hour (D) Half an hour

175. How many different people spoke at the meeting?

(A) Six (B) Three (C) Four (D) Eight

176. Which department did not receive any criticism during the briefing?

(A) Human Resources (B) Accounting (C) Sales (D) Design

177. What problem will K. Janes try to solve before the next quarterly meeting?

(A) Work is not being produced fast enough.

(B) Paperwork is being done too slowly. (C) More new employees need to be

hired. (D) Loyal customers are not being

satisfied.

I GO ON TO THE NEXT PAGE

Simulated TOEIC Test 1 Reading 393

2

3

4

5

6

Page 42: practical guide to the new toeic_practice tests.pdf

Questions 178-180 refer to the following letter.

@. fabulous Fhoto Agenc_y 7A, llF, Walker Building

297 Pimento Boulevard

San Anselmo, California 94960

Modem Business Office Supplies 29 Haight Street San Francisco, California 94150

November 5

Dear Sirs:

Our company, Fabulous Photo Agency, recently purchased several pieces of business office equipment from you. Our invoice is #3137, and we have paid the bill in full. After your movers set up our office, we noticed a few problems.

First of all, your movers bumped some of the file cabinets. Though this does not affect the units' usefulness, they do look aged already. Another problem witl1 the file cabinets is that you did not provide enough dividers to keep files separate.

One of the office desks is missing its keys. The invoice lists the desk as # 19201. The last problem is that one of the desk lamps is not functioning properly. We changed the bulb, but it still doesn't work right. That item is listed as #19249 on the invoice.

We have shown good faith in prepaying our bill. Please contact us at your earliest convenience to discuss how you will attend to the above-mentioned problems. We hope to do future business with you as our own grows. Thank you for your attention.

Yours sincerely,

H~~ Margaret Carnes Purchasing Manager Fabulous Photo Agency

394 Practical Guide to the NEW TOEIC® Test

Page 43: practical guide to the new toeic_practice tests.pdf

178. What is the main purpose of the letter?

(A) To point out fraud in advertising (B) To complain about damaged goods (C) To demand replacement of office

equipment (D) To request the firing of movers

179. What is wrong with the file cabinets?

(A) They have dents in them.

(B) They are not functional. (C) They lack proper lighting.

(D) They have no invoice.

180. What is wrong with one of the office

desks?

(A) It was dented in delivery.

(B) It has no dividers.

(C) It is too old to use.

(D) It has no keys.

I GO ON TO THE NEXT PAGE

Simulated TOEIC Test 1 Reading 395

2

3

4

5

6

Page 44: practical guide to the new toeic_practice tests.pdf

Questions 181-185 refer to the following article and letter.

Tech Magazine's Cell Showdown!

Model Price Internet Video MP3

Brokia Bgs $500 Yes Yes Yes Jamsung 3000 $350 No Yes Yes Todorola Lite $250 No Yes Yes

This year's cell phones feature a lot of style. However, if you were hoping for big advances in technology, you'll probably be disappointed.

Brokia Bgs: This phone will cost you an arm and a leg. However, for all that money, you do get a phone with Internet access, a video camera, and an MP3 player. Our recommendation: Buy last year's model, which has almost as many functions, and save your money.

Jamsung 3000: Trying to tell the difference between this phone and the Jamsung 2ooo was a challenge. Instead of being brightly colored, this one is black. Other than that, this is really just the 2000. Our recommendation: A lot of money for not a lot of phone. Don't bother.

Todorola Lite : This one doesn't have Internet or e-mail, either. However, it doesn't have the high price of the Jamsung, and it's got a great video camera and an MP3 player. Our recommendation: As long as you don't really need to surf the Net on your phone, this is the phone foryou.

Dear Tech Magazine:

I have a problem with the "Cell Showdown" article in your most recent issue. In particular, the author wrote that there was no new eel/phone technology this year.

In fact, this year's eel/phones have some important technological changes. A new kind of receiver was put in cel!phones this year that affects using the Internet on eel/phones. It doesn't change what you see on the screen, but it makes Internet access faster and more widely available.

Before you write an article saying there 's no new technology in something, you should get the truth first. I hope that you can write an article about this very interesting eel/phone technology in the near future.

Sincerely,

F~G~ Fiona Gray

- -~ -·· ---·

396 Practical Guide to the NEW TOEIC® Test

I

f

Page 45: practical guide to the new toeic_practice tests.pdf

181. What is the main purpose of the article? 184. Which phone does the article

(A) To convince people to buy the Todorola phone

(B) To debate the pros and cons of a few phones

(C) To declare which phone is the most stylish

(D) To describe big advances in phone technology

182. Why did Fiona Gray write a letter to the

magazine?

(A) To compliment the author (B) To request more information (C) To correct false information (D) To offer some advice

183. In the letter, the word "receiver" in paragraph 2, line 2, is closest in meaning to

(A) a device that sends information (B) a cellphone part that accepts data (C) a person who gets a new phone (D) a holder for a cellphone

recommend?

(A) Todorola Lite (B) Jamsung 3000 (C) Brokia B95 (D) All of the above

185. How is the most expensive phone different from the others?

(A) It can be purchased on the Internet. (B) It has a video camera and an MP3

player. (C) It allows the user to access the

Internet. (D) It doesn't have as many special

features.

I GO ON TO THE NEXT PAGE

Simulated TOEIC Test 1 Reading 397

2

3

4

5

6 -

Page 46: practical guide to the new toeic_practice tests.pdf

Questions 186-190 refer to the following letter and schedule.

TenyWesley Godric Computers Claremont, CA 92056

To Whom It May Concern:

I work for the Programming Department at Godric Computers, a local IT firm. We are searching for a location for our annual team building and planning event. We are considering renting one of the halls at the Anaheim Convention Center for this purpose. Factors that will play a part in our decision are cost,

/i

facility size, and technological amenities. Our seminar will be a three-day event I ' (Friday to Sunday) during a weekend in January. Please send a copy of your reservation schedule and a price list to the above address.

Regards,

T~W~ TenyWesley Godric Computers

Thank you for your interest in the Anaheim Convention Center. If you have any questions

about this information, call us at: 909-719-7401 .

January Convention Center 1{eservations Cafemlar

Guide: White: Unavailable*

*The Anaheim Convention Center only rents spaces in periods of 24 hours.

398 Practical Guide to the NEW TOEIC® Test

II

..

Page 47: practical guide to the new toeic_practice tests.pdf

186. What is the main purpose of the letter?

(A) To request information (B) To make a purchase (C) To give scheduling details (D) To announce an event

187. What information does Mr. Wesley NOT provide in his letter?

(A) The name of his company (B) The number of people who will

attend the seminar (C) The information he needs (D) The factors affecting his decision

188. Which of the following is an example of

an amenity?

(A) Free Internet access at the Convention Center

(B) A seminar on team building at an IT company

(C) Visiting the facility to check it out first

(D) Information provided by the Convention Center

189. Which hall of the Convention Center will fit Mr. Wesley's needs?

(A) HallA (B) Hall B (C) Hall C (D) None of the above

190. When will the Godric Computers event be if it is held at the Convention Center?

(A) January 15-17 (B) January 26-28 (C) January 20-21 (D) January 19-21

I GO ON TO THE NEXT PAGE

Simulated TOEIC Test 1 Reading 399

1

2

3

4

5

Page 48: practical guide to the new toeic_practice tests.pdf

Questions 191-195 refer to the following advertisement and application letter.

You may have dreamed of getting rich by working from home, but you probably

never thought it was really possible. There are many offers for get-rich-quick

schemes out there. Everyone knows that most of these are scams. Don't be fooled

by imitations. Trust ReaiBucks! We've been the most trusted name in work-from­

home contract services for over 20 years.

If you're interested in clerical work, translation, dictation, data entry, freelance

writing, and more, ReaiBucks is the company for you. In the Internet age,

companies don't need to have a lot of employees around full time. All they need

is someone with a computer who wants to earn some cash. That's why RealBucks

matches the most motivated people out there with the best companies. The

result: a lot of money going straight into your pocket.

Are you ready to make $1.000, $5,000, or even $10,000 per month without

leaving the comfort of your home? Let us help turn your dream into a reality. You

WILL make money with us. That's the ReaiBucks promise!

September 28

Dear RealBucks :

My name is Susan Granger . I read your advertisement in a magazine , and

I am writing to inquire about employment with RealBucks. I ' ll be happy

to send a complete resume . However , for the time being , here is some

information regarding education , work experience , etc .

I am 35 years old , and I have a secondary school diploma . I have some

college education from Northern Birmingham University , though I did

not complete my course of study there . During my time at university , I

majored in history .

Since leaving school , I have had extensive work experience in various

fields , including hospitality and guest relations . However , my main area

of occupation has been clerical work . Most recently , I have been working

as an executive assistant to the regional manager of a logistics firm

in Birmingham . Before this , I performed a number of t a sks at various

companies including filing , typing , data entry , and dictation .

Because of my work history , I feel that I am uniquely qualified for the

sorts of positions you have advertised . I am very eager to find work

through RealBucks and I hope that my qualifications and enthusiasm will

meet your requirements . Please -send me any and all information you have

regarding the current job openings you have and what the needs are for

these . Thank you in advance for your assistance .

Sincerely ,

Susan Granger

400 Practical Guide to the NEW TOEIC® Test

Page 49: practical guide to the new toeic_practice tests.pdf

191. What is being offered in the 194. In the advertisement, what does the

advertisement? word "age" in paragraph 2, line 2,

(A) A job with a computer company indicate?

(B) Jobs that people can do at home (A) Employees must be old enough to

(C) A dictation contest use the Internet.

(D) Protection from Internet scams (B) The Internet is becoming increasingly old .

;t 192. What is the main purpose of Susan (C) This time period is dominated by the (/j

Granger's e-mail? use of the Internet. -"""" (A) To send a resume to ReaiBucks (D) The Internet is helping people to

(B) To ask for a copy of an stay younger.

advertisement 1

(C) To express interest in marketing 195. Which job position has Susan Granger 2

(D) To describe her experience at held?

Real Bucks (A) Sales representative 3

(B) Hospital volunteer 4 193. What is Susan Granger's highest level (C) Business executive

of education? (D) Executive assistant 5

(A) High school diploma 6

(B) College degree (C) Postgraduate degree (D) Some teaching experience

I GO ON TO THE NEXT PAGE

Simulated TOEIC Test 1 Reading 401

Page 50: practical guide to the new toeic_practice tests.pdf

Questions 196-200 refer to the following brochure and e-mail.

Greenstreak is an international non-profit agency dedicated to the preservation of natural

habitats and the animals that rely on them. With offices in over 20 countries, Greenstreak

is one of the leading organizations in the fight to preserve our world's precious resources.

Since 1946, Greenstreak has worked with non-profit groups. world leaders in politics,

business and industry, and government organizations like the United Nations to change

the way people think about conservation.

As part of our mission, Greenstreak offers a number of courses for interested individuals

and groups of all levels on a range of topics related to ecology, biology, green living, and

conservation. We have a great deal of experience speaking to youth groups, schools,

university symposia, and government conferences. In the interest of spreading the

important message about honoring our planet, Greenstreak can customize a presentation

to the specific needs of any group. If your organization is interested in having Greenstreak

show up at your next event, please call your local Greenstreak office or visit us on the Web

at www.greenstreak.org.

[email protected]

events@greens treak.org

April13, 01:40.23 p.m.

Good morning! My name is Bart Manstead, and I'm a science teacher at Greenbrier Middle

School in Greenbrier, Alberta, Canada. We are arranging a special science fair for Earth Day

this year at the school. I'm currendy in the process of organizing guest speakers for the event,

and I would like to extend an invitation to Greenstreak to be our Guest of Honor.

We are a regional middle school for students in 5th through 8th grades (about 11-14 years old)

for the city of Greenbrier and a number of surrounding districts. We have approximately 3,500

students at the school.

In order to determine if Greenstreak can provide what we're looking for, I would appreciate

it if you could send me more information about your presentations. Obviously, any kind of

seminar would have to fit the needs of our students, the limits of our facilities, and the school

budget. However, we would be delighted to arrange for you to be present at our science fair.

Please reply at your convenience with the requested information. Thanks!

Bart Manstead

402 Practical Guide to the NEW TOEIC® Test

Page 51: practical guide to the new toeic_practice tests.pdf

196. What is Greenstreak promoting in their

advertisement?

(A) Presentations about environmental issues

(B) Job openings at over 20 offices worldwide

(C) An ecology conference at the United

Nations (D) A new series of Internet-based

courses

197. Why does Bart Man stead write to Greenstreak?

(A) His school is looking for a new science teacher.

(B) He wants Greenstreak to design a

science fair. (C) He is inviting Greenstreak to be

present at his school. (D) He wishes to be the Guest of Honor

at Greenstreak.

198. In the brochure, what does the word "green" in paragraph 2, line 3, refer to?

(A) inexperienced (B) the color of grass

(C) environmentally friendly (D) extremely jealous

199. How many different grades of students attend Greenbrier Middle School?

(A) 8 (B) 3,500 (C) 11-14

(D)4

200. What does Mr. Manstead request in his e-mail?

(A) A Greenstreak representative to come to the school

(B) More information on Greenstreak's presentations

(C) Information about Greenbrier Middle School

(D) A copy of a previous Greenstreak presentation

Stop! This is the end of the test. If you finish before time is called, you may go back to -Parts 5, 6, and 7 and check your work.

M·1·1:it·li:ij:i#3i:tWJ~

Simulated TOEIC Test 1 Reading 403

2

3

4

5

6 -

Page 52: practical guide to the new toeic_practice tests.pdf

404 Practical Guide to the NEW TOEIC® Test

Page 53: practical guide to the new toeic_practice tests.pdf

TEST 2 Questions

Part 1: Photographs

Part 2: Question-Response

Part 3: Short Conversations

Part 4: Short Talks

Part 5: Incomplete Sentences

Part 6: Text Completion

Part 7: Reading Comprehension

Page 54: practical guide to the new toeic_practice tests.pdf
Page 55: practical guide to the new toeic_practice tests.pdf

r-'1

~r = ;-; Q.

~ 0 trj .... ~

;3 ~ .... ~

"'*" 0 ......

~

TOEIC® Simulated Test

ANSWER SHEET

Part 1 Part 2 Part 3

No. ANSWER

ABCD No.

ANSWER

ABC No.

ANSWER

ABC No.

ANSWER ANSWER ANSWER ANSWER No. No. No.

ABC ABCD ABCD ABCD

z 1 ®®©@ 11 ®®© 21 ®®© 31 ®®© 41 ®®©@ 51 ®®©@ 61 ®®©@ 0 2 ®®©@ 12 ®®© 22 ®®© 32 ®®© 42 ®®©@ 52 ®®©@ 62 ®®©@ j:: v 3 ®®©@ 13 ®®© 23 ®®© 33 ®®© 43 ®®©@ 53 ®®©@ 63 ®®©@ w

"" "' 4 ®®©@ 14 ®®© 24 ®®© 34 ®®© 44 ®®©@ 54 ®®©@ 64 ®®©@ ~ 5 ®®©@ 15 ®®© 25 ®®© 35 ®®© 45 ®®©@ 55 ®®©@ 65 ®®©@ z w 6 ®®©@ 16 ®®© 26 ®®© 36 ®®© 46 ®®©@ 56 ®®©@ 66 ®®©@ t;; ::::i 7 ®®©@ 17 ®®© 27 ®®© 37 ®®© 47 ®®©@ 57 ®®©@ 67 ®®©@

8 ®®©@ 18 ®®© 28 ®®© 38 ®®© 48 ®®©@ 58 ®®©@ 68 ®®©@ 9 ®®©@ 19 ®®© 29 ®®© 39 ®®© 49 ®®©@ 59 ®®©@ 69 ®®©@ 10 (A) (B) (c) (D) 20 (A) (B) (c) 30 ®®© 40 ®®© 50 ®®©@ 60 ®®©@ 70 ®®©@

Part 5 Part 6 Part 7

ANSWER ANSWER ANSWER ANSWER ANSWER ANSWER ANSWER No. No. No. No. No. No. No.

ABCD ABCD ABCD ABCD ABCD ABCD ABC D

101 ®®©@ 111 ®®©@ 121 ®®©@ 131 ®®©@ 141 ®®©@ 151 ®®©@ 161 ®®©@ z 102 ®®©@ 112 ®®©@ 122 ®®©@ 132 ®®©@ 142 ®®©@ 152 i@@©@ 162 ®®©@ 0

6 103 ®®©@ 113 ®®©@ 123 ®®©@ 133 ®®©@ 143 ®®©@ 153 ®®©@ 163 ®®©@ w 104 ®®©@ 114 ®®©@ 124 ®®©@ 134 ®®©@ 144 ®®©@ 154 ®®©@ 164 ®®©@ "" "' 105 ®®©@ 115 ®®©@ 125 ®®©@ 135 ®®©@ 145 ®®©@ 155 ®®©@ 165 ®®©@ z 0 106 ®®©@ 116 ®®©@ 126 ®®©@ 136 ®®©@ 146 ®®©@ 156 ®®©@ 166 ®®©@ <t w 107 ®®©@ 117 ®®©@ 127 ®®©@ 137 ®®©@ 147 ®®©@ 157 ®®©@ 167 ®®©@ a:

108 ®®©@ 118 ®®©@ 128 ®®©@ 138 ®®©@ 148 ®®©@ 158 ®®©@ 168 ®®©@ 109 ®®©@ 119 ®®©@ 129 ®®©@ 139 ®®©@ 149 ®®©@ 159 ®®©@ 169 ®®©@ 110 I®®©® 120 I®®©@ 130 I®®©® 140 I®®©® 150 I®®©® 160 ®®©@ 170 ®®©@

TOEIC• IS a reg1stered trademart of the Educational Testmg Serv1ce (ETS) Tius pubhcallon IS not aflihated w1th, endorsed, or approved by ETS

Part 4

ANSWER ANSWER ANSWER No. No. No.

ABCD ABC D ABCD 71 ®®©@ 81 ®®©@ 91 ®®©@ 72 ®®©@ 82 ®®©@ 92 ®®©@ 73 ®®©@ 83 ®®©@ 93 ®®©@ 74 ®®©@ 84 ®®©@ 94 ®®©@ 75 ®®©@ 85 ®®©@ 95 ®®©@ 76 ®®©@ 86 ®®©@ 96 ®®©@ 77 ®®©@ 87 ®®©@ 97 ®®©@ 78 ®®©@ 88 ®®©@ 98 ®®©@ 79 ®®©@ 89 ®®©@ 99 ®®©@ 80 ®®©@ 90 ®®©© 100 ®®©@

ANSWER ANSWER IA.NSWER No. No. No.

ABCD ABCD ABCD 171 ®®©@ 181 ®®©@ 191 ®®©@ 172 ®®©@ 182 ®®©@ 192 ®®©@ 173 ®®©@ 183 ®®©@ 193 ®®©@ 174 ®®©@ 184 ®®©@ 194 ®®©@ 175 ®®©@ 185 ®®©@ 195 ®®©@ 176 ®®©@ 186 ®®©@ 196 ®®©@ 177 ®®©@ 187 ®®©@ 197 ®®©@ 178 ®®©@ 188 ®®©@ 198 ®®©@ 179 ®®©@ 189 ®®©@ 199 ®®©@ 180 I®®©© 190 I®®©® 200 I®®©©

Page 56: practical guide to the new toeic_practice tests.pdf

..

408 Practical Guide to the NEW TOEIC® Test

Page 57: practical guide to the new toeic_practice tests.pdf

LISTENING TEST

In the Listening test, you will be asked to demonstrate how well you understand spoken English. The entire Listening test will last approximately 45 minutes. There are four parts, and directions are given for each part. You must mark your answers on the separate answer sheet. Do not write your answers in your test book.

PART 1

Directions: For each question in this part, you will hear four statements about a picture in your test book. When you hear the statements, you must select the one statement that best describes what you see in the picture. Then find the number of the question on your answer sheet and mark your answer. The statements will not be printed in your test book and will be spoken only one time.

Sample Answer

Example

Statement (B) , "They're performing at a nightclub," is the best description of the picture, so you should select answer (B) and mark it on your answer sheet.

Simulated TOEIC Test 2 Listening 409

2

3

4

5

6

7

Page 58: practical guide to the new toeic_practice tests.pdf

1. ,,,.... ··-~·~~~·-·\"'-" ·~· ~-·. ' ·.~~ ! . ; .. # - - r.'* ~~ -.. '·~ .... '"" \ ' ·. .:

• • • ' 0 - ' ..A...,;. a -

2.

410 Practical Guide to the NEW TOEIC® Test

Page 59: practical guide to the new toeic_practice tests.pdf

3.

4.

GO ON TO THE NEXT PAGE

Simulated TOEIC Test 2 Listening 411

2

3

4

5

6

7

Page 60: practical guide to the new toeic_practice tests.pdf

5.

6.

412 Practical Guide to the NEW TOEIC® Test

...

Page 61: practical guide to the new toeic_practice tests.pdf

7.

8.

Simulated TOEIC Test 2 Listening 413

2

3

4

5

6

7

Page 62: practical guide to the new toeic_practice tests.pdf

9.

10.

\

414 Practical Guide to the NEW TOEIC® Test

Page 63: practical guide to the new toeic_practice tests.pdf

PART2

Directions: You will hear a question or statement and three responses spoken in English. They will not be printed in your test book and will be spoken only one time. Select the best response to the question or statement and mark the letter (A) , (B), or (C) on your answer sheet.

Example

You will hear: When does the next train arrive?

You will also hear: (A) In about five minutes.

(B) You can get there by bus.

(C) It's not far from here.

Sample Answer

e ®©

The best response to the question "When does the next train arrive?" is choice (A), "In about five minutes," so (A) is the correct answer. You should mark answer (A) on your

answer sheet.

11. Mali< your answer on your answer sheet. 26. Mali< your answer on your answer sheet.

12. Mali< your answer on your answer sheet. 27. Mali< your answer on your answer sheet.

13. Mali< your answer on your answer sheet. 28. Mali< your answer on your answer sheet.

14. Mali< your answer on your answer sheet. 29. Mali< your answer on your answer sheet.

15. Mali< your answer on your answer sheet. 30. Mali< your answer on your answer sheet.

16. Mali< your answer on your answer sheet. 31. Mali< your answer on your answer sheet.

17. Mali< your answer on your answer sheet. 32. Mali< your answer on your answer sheet.

18. Mali< your answer on your answer sheet. 33. Mali< your answer on your answer sheet.

19. Mali< your answer on your answer sheet. 34. Mali< your answer on your answer sheet.

20. Mali< your answer on your answer sheet. 35. Mali< your answer on your answer sheet.

21. Mali< your answer on your answer sheet. 36. Mali< your answer on your answer sheet.

22. Mali< your answer on your answer sheet. 37. Mali< your answer on your answer sheet.

23. Mali< your answer on your answer sheet. 38. Mali< your answer on your answer sheet.

24. Mali< your answer on your answer sheet. 39. Mali< your answer on your answer sheet.

25. Mali< your answer on your answer sheet. 40. Mali< your answer on your answer sheet.

I GO ON TO THE NEXT PAGE

Simulated TOEIC Test 2 Listening 415

1

3

4

5

6

7

Page 64: practical guide to the new toeic_practice tests.pdf

PART3

Directions: You will hear some conversations between two people. You will be asked to answer three questions about what the speakers say in each conversation . Select the best response to each question and mark the letter (A) , (B), (C) , or (D) on your answer sheet. The conversations will not be printed in your test book and will be spoken only one time.

41. Where most likely are the speakers? 44. Who is the woman talking to?

(A) At a doctor's office (A) Her boss

(B) In an office in Tokyo (B) Her boyfriend

(C) At the man's company (C) A friend

(D) At Dr. Carlton's house (D) A co-worker

42. Why is the man there? 45. What is the woman's problem?

(A) To book a flight to Tokyo (A) She has eye problems.

(B) To go to a meeting (B) She has no personal life.

(C) To change his appointment (C) The boss is disappointing her.

(D) To see a patient (D) She has had trouble with her

boyfriend.

43. What will the man probably do next?

(A) Leave and come back 46. What will happen if the woman's work

(B) Wait in the office doesn't improve?

(C) Fly to Tokyo (A) She'll be allowed to stay.

(D) Return on Thursday morning (B) She'll have personal problems.

(C) She'll lose her job. (D) She'll break up with her boyfriend.

416 Practical Guide to the NEW TOEIC® Test

Page 65: practical guide to the new toeic_practice tests.pdf

47. Who was Milly Peters?

(A) Mr. Daniels' cousin

(B) Their competitor's boss (C) A department manager (D) A newly hired employee

48. When was Milly Peters fired?

(A) After being caught looking through files

(B) As soon as she left the company (C) Before she resigned (D) After receiving a promotion

49. Why did Milly Peters lose her dedication

to the company?

(A) She had only been there for a few months.

(B) Paul started working for a competitor.

(C) Mr. Daniels has taken all of her files.

(D) She was refused a promotion.

50. What is George requesting?

(A) Permission to submit his report at a later time

(B) That the audit be postponed

(C) More information about the surprise

audit

(D) A pay raise

51. Why is Mrs. Smith reluctant to grant the

request?

(A) She is busy dealing with her own problems.

(B) She finds it hard to communicate with George.

(C) She doesn't think George is an asset to the company.

(D) It is not the first time that George has made this request.

52. What has Mrs. Smith decided to do?

(A) Cancel the audit immediately (B) Disclose no information to anyone (C) Speak to the head of the accounting

department (D) Extend the deadline of George's

report

53. When does the conversation take place?

(A) During a warehouse flood (B) Just before a warehouse flood (C) The day after a warehouse flood (D) The night of a warehouse flood

54. What does the woman want the man to do?

(A) Help clean the warehouse (B) Finish his sales reports (C) Dry out all the boxes (D) Get back to work

55. What will the man probably do next?

(A) Flood the warehouse (B) Hand in his reports at once (C) Move boxes for a little while (D) Stay upstairs

56. Why is the man talking to the woman?

(A) To buy tickets to a show (B) To get directions to the theater (C) To join the clean ing crew (D) To find a set of keys

57. Who might be able to solve the man's problem?

(A) The show manager (B) The woman (C) The cleaning crew (D) The actors

58. What will the woman probably do next?

(A) She will find the cleaning crew. (B) She will look for the man's keys. (C) She will call the man right away. (D) She will wait for the cleaning crew.

GO ON TO THE NEXT PAGE

Simulated TOEIC Test 2 Listening 417

2

4

5

6

7

Page 66: practical guide to the new toeic_practice tests.pdf

418 Practical Guide to the NEW TOEIC® Test

Page 67: practical guide to the new toeic_practice tests.pdf

PART4

Directions: You will hear some talks given by a single speaker. You will be asked to answer three questions about what the speaker says in each talk. Select the best response to each question and mark the letter (A), (B), (C), or (D) on your answer sheet. The talks will not be printed in your test book and will be spoken only one time.

71. What is the topic of today's conference 74. Where is this announcement being

presentation? made?

(A) Dunder-Miffling and VIP Marketing (A) At the train station (B) Sales and Marketing and (B) At a travel agency

Management Strategies (C) At the airport (C) Management Strategies and VIP (D) At a bus terminal

Marketing (D) Sales and Accounting 75. What information is given to the

passengers? 72. Where are the presentations being held? (A) The aircraft is permanently damaged.

(A) On the first floor (B) There are no flights to LA. (B) On the second floor (C) The flight is delayed for maintenance. (C) In the hotel lobby (D) Passengers need new travel (D) In the Green Ball Park arrangements.

73. What do employees need to enter the 76. What are passengers having connecting presentations? flights in Los Angeles told to do?

(A) Notebooks (A) Make alternate plans (B) Paper and pens (B) Proceed to the gate (C) Personal belongings (C) Listen for an announcement (D) Name tags (D) Get full refunds at the counter

I @·I·)~I(.IW:tWGIW~ /

Simulated TOEIC Test 2 1 Listening 419

;r;t (/) -N

1

2

3

5

6

7

Page 68: practical guide to the new toeic_practice tests.pdf

77. What is the purpose of the message? 83. What is the advertisement mainly about?

(A) To inquire about available positions (A) The new Super Customer card (B) To return an earlier phone call (B) A change in store hours (C) To inform Bill of a meeting time (C) The price of some products (D) To get in touch with an old colleague (D) The start of a sale

78. How does the speaker ask to be 84. What discount is offered in this special

contacted? sale?

(A) By fax (A) A 15% discount (B) By letter (B) A 35% discount (C) By e-mail (C) A 45% discount

(D) By telephone (D) A 50% discount

79. What will Stacy do on Tuesday? 85. Which customers can participate in the

sale? (A) Have an interview for a new job

(A) All customers in the store (B) Respond to Bill's message (B) Anyone who comes between 5 p.m. (C) Discuss paperwork with Bill

and 6 p.m. (D) Take over Bill's position (C) Those who see a sales clerk now (D) Those with a Super Customer card

80. What is being announced?

(A) New overtime hours 86. What is being announced? (B) Some recent accidents

(A) The attendants will start regular drink (C) A policy change

service. (D) The holiday schedule (B) The flight will be longer than

81. How many hours are workers limited to expected. (C) The plane has just taken off. each day? (D) The passengers can unfasten their

(A) 10 hours seatbelts. (B) 9 hours

87. For how long will the flight be delayed? (C) 8 hours (D) 7 hours (A) Until the plane is fixed

(B) Until they can use the runway 82. Why does the speaker make the (C) Until the airport opens again

announcement? (D) Until the passengers' seatbelts are

(A) Workers are taking too many breaks. all fastened

(B) There have been accidents recently. 88. What will the flight attendants do during (C) Employees are ignoring the old the delay?

policy. (A) Talk to the tower

(D) Employees demand fewer hours. (B) Circle the airport (C) Fasten their seatbelts (D) Serve drinks

420 Practical Guide to the NEW TOEIC® Test

Page 69: practical guide to the new toeic_practice tests.pdf

89. What is the purpose of the message? 95. Where is this talk most likely being

(A) To tell Gabe about new plans given?

(B) To reschedule a budget meeting (A) At a management conference (C) To answer a previous message (B) At a company picnic (D) To describe a design for a building (C) At a book fair

90. What does Carol say about Gabe's (D) At a training session

design changes? 96. Why must employees keep projects a ~ (A) They are good but expensive. secret? en -(B) They need to include extra features. (A) The company makes secret

.....,

(C) She has lots of problems with them. airplanes. (D) She cannot see them until Thursday. (B) The military is watching the company. 1

91. What does Carol suggest they do? (C) The projects are illegal.

2 (D) The company is a secret. (A) Make plans for a new design 3 (B) Include more extra features 97. What will happen if employees reveal

(C) Go to GloboBank on Thursday secret projects?

(D) Meet to talk about project costs (A) Their contracts will be signed. 5 (B) The company will classify them. (C) They might be fired and go to jail.

6

92. Where is the speaker? (D) They will follow the rules. 7

(A) In a car repair shop (B) At a car show (C) On a race track 98. Where would the report most likely be

(D) Inside a car museum heard?

93. What information is included in the talk? (A) In an economics classroom (B) At a children's toy store

(A) A description of some Chester (C) On a news program vehicles (D) At a farewell party

(B) A plan for the Chester Motors factory tour 99. How many of the country's toys have

(C) The names of a few vehicle problems?

engineers (A) 1,500 (D) Details of the process of building (B) Half of them

ships (C) 483

94. What are listeners expected to do if they (D) One third of them

have questions? 100. What did government officials say about

(A) Ask the speaker the news?

(B) Use their imaginations (A) All toy factories need to be inspected. (C) Visit the design studio (B) More control on imports is needed. (D) Tour the factory (C) Safety guidelines should be lowered.

(D) Toys from other countries should be

banned .

This is the end of the Listening test. Turn to Part 5 in your test book.

Simulated TOEIC Test 2 Listenin 421

Page 70: practical guide to the new toeic_practice tests.pdf

READING TEST

In the Reading test, you will read a variety of texts and answer several different types of reading comprehension questions. The entire Reading test will last 75 minutes. There are three parts, and directions are given for each part. You are encouraged to answer as many questions as possible within the time allowed.

You must mark your answers on the separate answer sheet. Do not write your answers in your test book.

PARTS

Directions: A word or phrase is missing in each of the sentences below. Four answer choices are given below each sentence. Select the best answer to complete the sentence. Then mark the letter (A) , (B), (C), or (D) on your answer sheet.

101. Our proposal may not be 1 00 percent 103. Mary Parker Follett, a noted social satisfaction guarant7ed, but it------- us worker in the early 20th century, -------

almost two weeks to work it out, and I management as the art of getting things

believe it works. done through people.

(A) took (A) confined

(B) spent (B) refined

(C) cost (C) defined

(D) needed (D) fined

102. Although Apple has not disclosed how 104. The CEO is now faced with a decision

many iPhones were -------- at launch on whether to downsize the current

Friday, analysts expect stocks will sell operations or lay off ------- 500

out by early next week. employees.

(A) useful (A) addition

(B) available (B) an addition of

(C) replaceable (C) an additional

(D) inevitable (D) in addition

422 Practical Guide to the NEW TOEIC® Test

Page 71: practical guide to the new toeic_practice tests.pdf

105. It is encouraging to find that more 109. This handout will help you write

and more people are getting ------- in business letters in many different

preserving the ecological system of the situations ranging from applying for a

rainforests of the Amazon. job to ------- information.

(A) interest (A) request

(B) interested (B) requesting

(C) interesting (C) requests ;i (D) interests (D) requested (/J -106. Paul is a man ------- I believe has the 110. To deal with the ever-increasing number ~

best understanding of what it is that of tourists from Europe, the shuttle

makes our products different from transport company has decided to

others on the market. provide ------- bus service on holidays. 2

(A) who (A) often

(B) whom (B) really 3

(C) whose (C) frequent 4 (D) how (D) many

107. By teaming up, we finally solved the 111. Whether we can succeed in meeting the 6 thorny problem that might ------- never targeted costs ------- on how hard we try

7 have been solved by one person alone. to budget our money.

(A) solely (A) depends

(B) fully (B) depend

(C) further (C) depended

(D) otherwise (D) depending

108. We're happy to announce that this 112. The newspaper was sharply -------- of year's financial report is ready to be the false account the general manager

------- to management for their reference gave about the financial situation of his

at the meeting this afternoon. company.

(A) distributed (A) critic

(B) disposed (B) criticism

(C) prescribed (C) criticize

(D) deducted (D) critical

Simulated TOEIC Test 2 Reading 423

Page 72: practical guide to the new toeic_practice tests.pdf

113. A few days ago that newspaper ran 117. All CEOs need -------that it pays to an editorial which spoke ------ of those introduce an energy efficiency prowam who took the issue of global warming that can help save their business seriously. money.

(A) widely (A) convince (B) highly (B) to convince (C) deeply (C) to be convinced (D) broadly (D) be convincing

114. ------- all the novels Mr. Gordon 118. In a------- conducted by the research Samuelson wrote, The Fishing Village , team, 94 percent of the consumers which was released two months ago, is agreed that the Internet presents a without doubt the best I've ever read . threat to children .

(A) On (A) survey (B) Of (B) design (C) Between (C) broadcast (D) Through (D) scandal

115. As the owner or potential owner of a 119. Since we have so -------time left, I small business, ------- will come a time suggest we should not do anything when you need more capital, so you'll about the financial plan until tomorrow, start thinking about borrowing money. when we can have more time to review

(A) it it.

(B) that (A) short (C) there (B) little

(D) you (C) brief (D) small

116. Advertisers are always trying to find ways of attracting the attention of as 120. Local leaders are delighted at the many consumers as -------. prospect of hundreds of highly skilled

(A) likely jobs and training opportunities -------

(B) probable created in the area.

(C) possible (A) being

(D) he can (B) is (C) are (D) have

424 Practical Guide to the NEW TOEIC® Test

Page 73: practical guide to the new toeic_practice tests.pdf

121. Trustus Inc. is the largest firm of 125. We ordered dozens of student business brokers in South America that desks from you on July 1 with the offers a large ------- of franchises or understanding ------- they would be businesses for sale. delivered within a week.

(A) amount (A) what

(B) quality (B) where

(C) volume (C) that a;f (D) selection (D) which tn ....

122. Before buying a factory ------- setting 126. David is a nice guy who used to be a N

up one of your own in that area, you co-worker of------- when I was living should carefully evaluate the security and working in the Middle East about risks there. five years ago.

2 (A) so that (A) I (B) nor (B) me 3

(C) or (C) myself 4 (D) so (D) mine

123. Organically grown greenhouse 127. It is ------- to find that workers who make 6 vegetables are seeing increased sales the most money are often those who

7 among middle-income consumers ------- are the least content with their jobs. their higher cost. (A) surprising (A) although (B) surprised (B) because (C) surprise

(C) despite (D) surprises (D) even if

128. Business has been so good over the 124. We ------- the financial report yesterday, last two seasons ------- the general

but the boss asked us to collect manager has promised to take all the information for a briefing he's supposed staff on a trip to Europe in early July. to make for some new employees. (A) because (A) should finish (B) before (B) had finished (C) that (C) should have finished (D) therefore (D) were finished

GO ON TO THE NEXT PAGE

Simulated TOEIC Test 2 Reading 425

Page 74: practical guide to the new toeic_practice tests.pdf

129. To help us conduct the survey faster 133. It has been proven that wooden doors and more efficiently, please submit have a -------to expand in humid your three favorite choices in ------- weather unless they are treated with a of preference via the e-mail address chemical stabilizer. below. (A) move (A) order (B) direction (B) ordering (C) tendency (C) ordered (D) desire (D) orderly

134. Those taking part in the ultra marathon 130. In------- of emergency, all hotel guests in Arizona next month ------- to carry

are reminded to use the emergency a first-aid kit, which should include a stairs rather than the elevator. remedy for snakebites.

(A) case (A) advise (B) event (B) advised (C) situation (C) be advised (D) condition (D) are advised

131. Considering the high turnover rate of 135. Weather-------, the morning activity will entry-level secretaries, the general be a hike in the mountains or, for those manager has decided to ------- their who are interested in fine art, a visit to base salaries. the museum.

(A) rise (A) permit

(B) raise (B) permitted

(C) arise (C) permits

(D) arouse (D) permitting

132. CHANTIX is the ------- prescription 136. As one of our new recruits, you will medicine this company successfully be required to ------- in an orientation

developed half a year ago to help adults session that is scheduled to start this

quit smoking. coming Thursday.

(A) late {A) investigate

(B) later (B) participate

(C) latter (C) candidate

(D) latest (D) subordinate

426 Practical Guide to the NEW TOEIC® Test

Page 75: practical guide to the new toeic_practice tests.pdf

137. ------- yourself with all the company guidelines by the end of the third day of training, as there will be a test on the final day.

(A) Familiarize (B) Familiarizing (C) To familiarize

(D) Famil iar

138. Having used up her sick leave while in the hospital , Mary had no option but ------- personal leave instead.

(A) using (B) use (C) to use (D) by using

139. The purpose of this system is -------trade flow as freely as possible, making sure that there are no undesirable side-effects.

(A) help (B) to help (C) helping

(D) in helping

140. The proposal , ------- I agree, is unfortunately not accepted by my partner, so I'm afraid you'll have to come up with another one.

(A) that (B) which (C) to which (D) in that

I GO ON TO THE NEXT PAGE

Simulated TOEIC Test 2 Readin g 427

~ t.n .... N

1

2

3

4

6

7

Page 76: practical guide to the new toeic_practice tests.pdf

PART6

Directions: Read the texts that follow. A word or phrase is missing in some of the sentences.

Four answer choices are given below each of the sentences. Select the best answer to

complete the text. Then mark the letter (A), (B), (C) , or (D) on your answer sheet.

Questions 141-143 refer to the following passage.

Over the last ten years, the number of people entering managerial roles ------- increased

141. (A) was

(B) were

(C) has

(D) have

significantly. Managers in virtually all kinds of industries are assuming increasing responsibility while still ensuring that they ------- their routine requirements.

142. (A) meet

(B) deny

(C) search

(D) look

The role is expanding but is often made more difficult because of the lack of management

training received. A recently retired management consultant once said , "Most managers ar~

promoted because of their expertise. And it is probable that for every one manager who has

received professional training there are ten who've had no formal training." It is inevitable that

managers are faced with many issues and problems ------- a daily basis. A professor who also

143. (A) for

(B) on

(C) with

(D) through

heads a leading management consulting company argues that organizations seek to recruit

talented individuals without realizing that talent can be difficult to manage. Managers have to work

with a range of people: the talented, the team players, those who handle stress well and those

who don't.

428 Practical Guide to the NEW TOEIC® Test I

Page 77: practical guide to the new toeic_practice tests.pdf

Questlo,.s 144-146 refer to the following memorandum.

MEMORANDUM

TO: Social Work Students FROM: Dr. Robert Lang SUBJECT: Student Body Demographic Profile

The Social Work Department has implemented a program monitoring system which ------- data from a variety of sources. The purpose of this ongoing evaluation is to improve our 144. (A) includes

(B) including (C) inclusive (p) include

department's efforts to meet program objectives and to make program changes based upon

information received .

We request your participation in our efforts to understand our students' demographic profile, and to obtain ·evaluations pertaining to their work. We -------your assistance in this effort

145. (A) experience (B) appreciate (C) undermine (D) avoid

and strongly believe this approach to program evaluation will in the long run prove a valuable asset to our department and program.

------- you have any questions or concerns, please feel free to contact me. 146. (A) Would

(B) Could (C) Might (D) Should

Thank you for your help.

I GO ON TO THE NEXT PAGE

Simulated TOEIC Test 2 Reading 429

2

3

4

Page 78: practical guide to the new toeic_practice tests.pdf

Questions 147-149 refer to the following advertisement.

ADMINISTRATIVE ASSISTANT WANTED

Goodwill Disability Care Service requires an administrative assistant to work in its central New York office. The successful ------- will enjoy working as part of a dedicated team, with the

147. (A) serviceman (B) missionary

(C) candidate (D) janitor

added satisfaction of working for an organization committed to the care and support of disabled people.

The job involves a variety of administrative duties. These will include filing, letter writing , and other secretarial work. Training will be provided, if necessary, to es:luip the job-holder ------- computer skills to enable them to use the o,rganization's computer system. 148. (A) on

(B) with (C) for (D) about

This position is ------- on a full-time basis only. For further information, please contact us 149. (A) available

(B) useful (C) informative (D) temporary

at (315) 336-2100.

430 Practical Guide to the NEW TOEIC® Test

Page 79: practical guide to the new toeic_practice tests.pdf

Questions 150-152 refer to the following e-mail.

To: [email protected] From: [email protected] Subject: Sales Meeting

For the last three quarters, our sales have dropped nearly 30 percent. We understand that there is no problem with the quality of the new products recently released, so I believe our sales strategies do ------- something to be desired, and we cannot make excuses. Seeing the

150. (A) leave (B) allow (C) let (D) get

first drop should have motivated us to think up better sales strategies and new ideas of how we can increase revenue.

Unfortunately, we have yet to really work towards improving our financial year so far. I, therefore, would like to get you all to------- feasible ways to help improve the current situation.

151. (A) run into (B) come up with (C) hold up (D) make up for

Overall , our company has been in a slump for the last year. For this reason, I demand that a meeting -------this Friday at 2 p.m. Please be on time with your proposals.

152. (A) will be held (B) to be held (C) held (D) be held

GO ON TO THE NEXT PAGE

Simulated TOEIC Test 2 Reading 431

1

2

3

4

5

Page 80: practical guide to the new toeic_practice tests.pdf

PART7 Directions: In this part you will read a selection of texts, such as magazine and newspaper articles, letters, and advertisements. Each text is followed by several questions. Select the best answer for each question and mark the letter (A), (B) , (C) , or (D) on your answer sheet.

Questions 153-154 refer to the following schedule.

· • Porter and Willis Accounting Seminar

Saturday, November 18

Porter and Willis, one of the m ost prestigious accounting firms in Wisconsin, is offering a one-day seminar for senior accountants and accounting managers. This seminar, limited to 40 participants, will be held at the office of Porter and Willis on the 10th floor of the Bates Building, 231 Racine Boulevard, Racine, Wisconsin. Applicants may register in person from Monday to Friday, November 13-17, or online anytime at www.porterandwillis.com. T he application fee is US$150, inclusive of lunch. A dinner at 6:30p.m . at the Sheraton Hotel across the street will be offered for an additional US$ 50.

Saturday, November 18

0900

1000

1100-1300

1300

1400

1500

Keynote Speech Rev isiting the Accounting Equation Dr. Philip Williams, D epartment of Accounting, University of Wisconsin, Madison

Keynote Speech

Chapter 11 and the Accountant Ms. Jeanette Raleigh, Senior Partner, Masters and Thompson

Lunch

Bankruptcy and Pension Provisions Mr. Allen Walker, Cooper Trust

Computer Software and the Accountant: Updating Our Proficiencies Mr. Gerhard Stoffen, Tesco Retailing International

Challenges in the Revised Wisconsin State Tax Code Ms. Allison Corder, Wisconsin Bureau of Accounting

For further information, contact Ms. Peggy Nunan at Porter and Willis

(262) 211-8989, ext. 20.

432 Practical Guide to t he NEW TOEIC® Test

Page 81: practical guide to the new toeic_practice tests.pdf

153. How much must a participant pay for the seminar and dinner?

(A) $100

(B) $150

(C) $200 (D)$250

154. Which speech would a specialist on retirement accounting most likely be

interested in?

(A) Revisiting the Accounting Equation

(B) Chapter 11 and the Accountant (C) Bankruptcy and Pension Provisions (D) Challenges in Revised Wisconsin

State Tax Code

GO ON TO THE NEXT PAGE

Simulated TOEIC Test 2 Reading 433

2

3

4

5

Page 82: practical guide to the new toeic_practice tests.pdf

Questions 155-157 refer to the following e-mail message.

All employees ([email protected])

Dave Spears ([email protected])

FYI- Company Soccer Match

All employees who will be participating in the company soccer match against

lntercomtech should be aware that the time of the game has changed from 4:00

p.m. to 4:30p.m. As a result, the shuttle bus that will be taking players to the

field will leave 30 minutes later, at 3:45p.m. rather than 3:15p.m. Make sure

that you arrive at least 1 0 minutes early to the south entrance to allow time to

sign out and to board the bus. After the bus arrives at the playing field, there

should be about 30 minutes to warm up before the game begins.

Spectators of the game must provide their own transportation. We hope that

everyone who is not playing will attend and cheer their co-workers on. Let's

have lots of fans there to celebrate our team's success. Thanks!

Dave Spears

Employee Events Coordinator

155. What is the main purpose of this e-mail?

(A) To announce an upcoming soccer game

(B) To inform people of a schedule

change (C) To promote company spirit (D) To explain the schedule for the new

buses

156. Who is the intended recipient of this e-mail?

(A) The entire staff of the company (B) Employees having to work

overtime (C) Soccer fans who will attend the

game (D) Workers who will be riding the bus

157. When will the bus arrive at the soccer field?

(A) 1 0 minutes early (B) At 4:30 p.m. (C) 30 minutes before the game (D) At 3:45p.m.

434 Practical Guide to the NEW TOEIC® Test

Page 83: practical guide to the new toeic_practice tests.pdf

Questions 158-159 refer to the following advertisement.

Not surprisingly, a recent study says that over 85% of business travelers feel that they could

use some help finding ways to relax and enjoy their international travels. No one does this

better than GlobaiServe, Inc. GlobaiServe offers comprehensive leisure services for people on

the go around the globe, including transportation, reservations, tickets, and even assistance

with currency and traveler's checks. You don't have time to waste in finding your way around

an unfamiliar city in an unfamiliar country. On your next business trip, let GlobaiServe's team

of experts put together a package that will finally allow you to enjoy international business

travels. Contact the GlobaiServe International Call Center today to speak with one of our

representatives, and let us help you see the world, stress-free.

158. What kind of company is GlobaiServe, Inc.?

(A) A news agency (B) A travel service (C) A shuttle service (D) An airline company

159. Which would be the most likely customer of GlobaiServe, Inc.?

(A) An American family traveling to Japan

(B) An executive driving from New York to Los Angeles

(C) A company opening a new branch overseas

(D) A manager attending a meeting in another country

Simulated TOEIC Test 2 Reading 435

2

3

4

5

Page 84: practical guide to the new toeic_practice tests.pdf

Questions 160-162 refer to the following e-mail message.

Software Engineers ([email protected])

IT Support ([email protected])

Monitoring Software Installation

Aug. 15, 2:43 p.m.

This e-mail is to notify you about an important upcoming change related to

the computer use policy. As of next week, activity-monitoring software will be

installed on all of your computers. This software will track which programs are

used and which websites are visited. The information will include only the names

of the programs, addresses of the websites, and times at which you were using or

visiting them.

This is being done in response to recent concerns about the activities and time

management of several employees. The information gathered will be used to

ensure that you comply with the computer use agreement that you all signed

when you came to work here. Employees found in violation of the agreement will

be reprimanded and punished in keeping with company regulations. If you have

any questions or comments, please direct them to your supervisors.

436 Practical Guide to the NEW TOEIC® Test

Page 85: practical guide to the new toeic_practice tests.pdf

160. What is the main topic of this e-mail?

(A) Replacement of a few old computers (B) Some new computers that will be

purchased (C) New software that will be installed (D) Two staff members being punished

for policy violations

161. Why was the e-mail written?

(A) To help the employees gather

information (B) To inform workers of a meeting (C) To ensure employees are following

the policy (D) To allow employees to monitor each

other

162. What should employees do if they want

to complain?

(A) Contact their supervisors (B) Reply to the e-mail (C) Delete the new software (D) Sign the computer use policy

Simulated TOEIC Test 2 Reading 437

1

2

3

4

5

6 -

Page 86: practical guide to the new toeic_practice tests.pdf

Questions 163-166 refer to the following letter.

WHEELING STEEL

35 Major Drive Wheeling, West Virginia 26003

Phone: 304·229·1177 www.wheelingsteel.com

Dear Customer: June 30, 2007

Beginning January 1, 2008, we will discontinue a number of our items in our

product lines. Please note the following:

Ball bearings CX-100, CX-200, and CY-450 series

Rolled sheeting Galvanized iron, 25-millimeter

thickness Zinc-coated steel,

35-millimeter thickness

Tubing Copper-plated, 2.5-centimeter diameter

Titanium-reinforced, 3-centimeter

diameter

Square plates ZZ-115 and ZZ-118 series

Any orders for the above items received in our offices before January 1, 2008 will be honored. Thereafter, we will continue to sell only unsold stock. Contact your

nearest Wheeling Steel Field representative with any questions. We regret any

inconvenience to our customers.

163. What is the purpose of this letter?

(A) To inform customers of increased prices

(B) To inform customers of product changes

(C) To inform customers of a new factory

(D) To inform customers of new field personnel

164. Which of the following means of communication with Wheeling Steel is NOT available?

(A) Phone (B) Fax (C) Mail (D) The Internet

Sincerely yours,

set Pctttejl\, Ed Patten

Manager, Sales and Service

165. Who would most likely do business with Wheeling Steel?

(A) Software companies (B) Real estate brokers (C) Automotive companies (D) Travel agencies

166. What will happen before January 1, 2008?

(A) Prices will go down. (B) Prices will increase. (C) Several products will be

discontinued. (D) New orders will be accepted.

438 Practical Guide to the NEW TOEIC® Test

Page 87: practical guide to the new toeic_practice tests.pdf

Questions 167-170 refer to the following document.

HAPPY HOMES PET RENTAL (HHPRJ

Wilton, Minnesota

Date: November 21, 2007 Renter: Betty Savage Pet Rented: Skippy-female beagle# 613-947382

By signing this contract, the above named renter agrees to follow all of the rules outlined below and agrees to pay for any injuries or illnesses that occur as a direct result of the renter's actions.

• All pets must be picked up and dropped off on time. Thirty minutes is allotted for late pick-ups before pets become available to other

clients. Late drop-offs will be charged $100 for each hour overdue.

• All pets must be fed the foods I treats list ed on the Food Handout.

• All pets must be transported and housed in the carriers I kennels I cages

provided by H H PR.

• All provided supplies must be ret urned during drop-off. Renters will be

fined for missing or damaged supplies.

Renter's Signature: lSettkj SC!VC!ge

HHPR: l>tive MiUer, 'MA-n.Affer

167. What is this contract for?

(A) Adopting a pet (B) Giving away a pet (C) Renting a pet (D) Buying a pet license

168. In which situation would Betty have to pay medical bills?

(A) She drops Skippy whi le carrying it. (B) The dog won't eat and becomes

weak. (C) She forgets the pet shampoo at

home. (D) She drops Skippy off two hours late.

169. Who is Olive Miller?

(A) Skippy's owner (B) A client of HHPR (C) Betty Savage's lawyer (D) A manager at HHPR

170. What will happen to a pet if not picked up within 30 minutes?

(A) The pet will be delivered by HHPR. (B) It will become available to others. (C) The client will be charged $100. (D) HHPR will get rid of the pet.

Simulated TOEIC Test 2 Reading 439

1

2

3

4

5

6 -

Page 88: practical guide to the new toeic_practice tests.pdf

Questions 171-173 refer to the following letter.

Stock Student Loans

103 Hibicus Street Kansas City , KS 66101

Telephone : (913) 573 - 5721

Mr . Peter Fry 8495 River Road , Apt . 3C Leawood , KS 66206

December 16

Dear Mr . Fry :

This is a reminder that your Stock Student Loan (Account RAF8594022958) is paid in advance . Therefore , no payment is due at this time . However , we would like to remind you that interest on your loan accrues daily , and it is in your best interest to continue making your monthly payments in advance or on time .

We would also like to take this opportunity to inform you of our new Easy Pay system . This method of payment automatically makes your monthly loan payment for you by directly transfe r ring funds from a savings/checking account each month . Enclosed is a brochure highlighting all of the advantages the Easy Pay system offers Stock Student Loans clients . Look inside for information on how to sign up for this payment option .

If you have any questions about your loan , payment , or interest rates , please contact repres~ntatives .

Sincerely , J . P . Burgerburg

Director of Student Loan Services

440 Practical Guide to the NEW TOEIC® Test

methods of one of our service

II

I

:

Page 89: practical guide to the new toeic_practice tests.pdf

171. What is the letter mainly about?

(A) Loan payment (B) Renting cars (C) Savings accounts (D) Traveler's checks

172. What kind of loan does Peter Fry have?

(A) A homeowner's loan (B) A small business loan (C) A student loan (D) A car loan

173. What does the Easy Pay system offer?

(A) Loan transfers to other companies (B) New savings I checking accounts (C) Automatic monthly payments (D) Automatic cash withdrawals

I GO ON TO THE NEXT PAGE

Simulated TOEIC Test 2 Reading 441

2

3

4

5

6 -

...

Page 90: practical guide to the new toeic_practice tests.pdf

,

Questions 174-176 refer to the following advertisement.

Travel Plan Plus is having its annual travel package sale! For amazingly low prices, you

can take a weeklong vacation to any of the beautiful destinations listed below!

~ Act fast! This offer ends on May 31 st!

Package I: J.lawaii, USA $1,&'00 On the beautiful island of Oahu, you'll stay at a complete spa and resort. Enjoy

horseback riding, nature trails, and a private beach. Contact one of our agents to find

out how you can win a full two-week package for the same price!

Package 2: Munich, Germany $2,100 The historic town of Munich waits for you! Start your trip by touring the town and visiting

some of the world-famous museums.

Package 3: Sydney, Australia $&'50 No trip to Sydney would be complete without a stop at the Sydney Opera House.

Also, you'll climb a bridge and trek into the outback. Get an up-close experience with

Australia's most beloved animal, the koala.

For further information, call a Travel Plan Plus agency or check online at www. travelplanplus.com/discounts.

Don't wait! Plan your dream vacation today!

442 Practical Guide to the NEW TOEIC® Test

Page 91: practical guide to the new toeic_practice tests.pdf

174. Which package is best if cost is a major concern?

(A Package 1 (B) Package 2 (C) Package 3 (D) All of the above

175. According to the advertisement, what is NOT included as part of the Sydney package?

(A) A visit to the Sydney Opera House (B) A bridge-climbing endeavor (C) A spa massage (D) A hiking trip

176. How can one find more information about the travel package sale?

(A) Through TV commercials (B) Through an agent or online (C) By replying to this message (D) By sending an e-mail

Simulated TOEIC Test 2 Reading 443

1

2

3

4

5

6

Page 92: practical guide to the new toeic_practice tests.pdf

Questions 177-180 refer to the following article.

A major breakthrough was announced in the computing world yesterday

as a new type of processor was released by the computing firm Smart Designs,

Inc. According to Smart Designs, the new processor uses special technology to

massively decrease the size of its components, allowing the power of as many as

100 top-of-the-line processors to fit into the space of one. While details of the new

technology have not been released yet, a demonstration of a computer using the

new processor was made yesterday at Compucon, an annual New York computer

convention.

Founded by entrepreneur Jamie Smart in 1987, Smart Designs originally

manufactured a range of computer products before focusing on their processor

designs in the late 90s. It was announced that Smart, now CEO, will give a press

conference later this week to describe plans for the production and sale of the new

computer component, as well as more details of the advanced technology it relies

on. However, analysts are already forecasting a price tag of over $2,500 per unit

based on the demonstration at Compucon. Shares of Smart Designs, Inc. (NYSE:

SMOG) have risen by 33 percent since the news was released.

444 Practical Guide to the NEW TOEIC® Test

,

Page 93: practical guide to the new toeic_practice tests.pdf

177. Which is the best title for this article?

(A) Entrepreneur Founds Computing Firm

(B) News Conference Announced to Name CEO

(C) Computing Stock Soars to $2,500

(D) High-Powered Computer Unveiled

178. Where was the new technology shown

to the public?

(A) At a New York computer company (B) At a technology convention (C) At a welcoming party (D) At a meeting of shareholders

179. What was a result of the release of the

new computer part?

(A) Jamie Smart was made CEO of the company.

(B) Analysts valued the company at

$2,500.

(C) Smart Designs, Inc. stock rose by

33 percent.

(D) 100 top-of-the-line processors were

built.

180. When was the new technology first announced?

(A) The day before the article was written

(B) During the late 90s

(C) In 1987

(D) Later this week

I GO ON TO THE NEXT PAGE

Simulated TOEIC Test 2 Reading 445

2

3

4

5

6 -

Page 94: practical guide to the new toeic_practice tests.pdf

Questions 181-185 refer to the following recipe and letter.

61arge eggs

1 cup and 1 tablespoon sugar

1 /2 teaspoon vanilla extract

1/4 teaspoon ground nutmeg

3/4 cup brandy

1 /3 cup dark rum

2 cups whipping cream

2 cups milk

All liquids should be very cold . Refrigerate in advance.

Beat the eggs for 2 or 3 minutes with an electric mixer at medium speed until very frothy. Gradually beat in the sugar, vanilla, and nutmeg. By hand, stir in the cold brandy, rum, whipping cream. and milk. Chill before serving. Sprinkle individual cups with more nutmeg.

Dear Ms. Daniels:

Thank you for your inquiry about our recipe page. It's always great to hear from our

magazine's readers. Here's what we have to say in response to your letter.

First of all, we suggest that you always consult a doctor and do your own research before eating anything you find questionable. That said, people have been eating raw eggs for years with only a handful of sickness cases. One thing to keep in mind before consuming

any is how fresh your eggs are. It is advised that you only consume very fresh raw eggs.

Otherwise, you might be asking for trouble.

As for your other question, sherry is a suitable replacement for rum or brandy when making eggnog. However, it's best to test how well the flavor blends with the other ingredients before preparing a batch for guests. It is possible some of the other spices

could interfere with the sherry's original flavor.

Sincerely,

J¥-~4 Joyce Billows Editor in Chief

446 Practical Guide to the NEW TOEIC<Il Test

Page 95: practical guide to the new toeic_practice tests.pdf

181. According to the recipe, what needs to 184. How does the editor feel about eating

be done in advance? raw eggs?

(A) The liquids must be heated. (A) It is never a good idea.

(B) The refrigerator must be turned on. (B) Everyone ought to eat them.

(C) The eggs must be boiled and (C) It is each individual's choice.

peeled. (D) They should be eaten with garlic

(D) You should have all the ingredients bread. ;4

ready and chilled in advance. 185. What is the editor's concern with using

en ..... 182. How do the eggs become frothy? sherry?

N

(A) By using an electric mixer (A) Guests don't like waiting for it.

(B) By stirring them by hand (B) It will make the eggnog too spicy. 1

(C) By chilling them before serving (C) The flavors might not blend well. 2

(D) By frothing them in individual cups (D) Sherry is much more expensive than rum or brandy. 3

183. What is the purpose of Ms. Daniels' 4

letter? 5 (A} To complain about an article (B) To submit an eggnog recipe (C) To ask questions about a recipe (D) To answer questions about eggnog

I GO ON TO THE NEXT PAGE

Simulated TOEIC Test 2 Reading 447

Page 96: practical guide to the new toeic_practice tests.pdf

Questions 186-190 refer to the following billing statement and notice.

Prairie Gas Company

147 Redfield Place Alliance, Nebraska 69301

Phone: (308) 947-1111 Fax: (308) 947-1122 www.prairiegasco.com

To: Duvall Manufacturing Company 27 Arterial Highway Alliance, Nebraska 69301

Period From

Gas Bill

1 September, 07

Consumption (December monthly total)

1405 hectoliters heating gas

Last meter reading: 5 September, 07 Current meter reading: 3 October, 07 Next scheduled meter reading: 4 November, 07 Mailed out: 10 October, 07 Payment due: 25 October, 07

To

30 September, 07

Charge

$10,459.45

Attention All Employees Due to the unusual cold snap in December, our gas bill for the month was the highest

on record. In anticipation of further cold weather in January and February, Duvall Manufacturing is instituting the following energy conservation measures immediately until further notice:

All offices, storage rooms, and warehouses will lower their thermostats from 77•F to 74•F during working hours. After hours, all managers are responsible for reducing the temperature further to 70.F. Upon starting work, managers should raise thermostats to 74·F. This small measure is estimated to save the company at least $400 per month. The company advises the wearing of a sweater or vest to work.

All doors and windows must be secured after working hours in offices. This will reduce any unwanted drafts from further cooling buildings.

The factories will set thermostats to 70•F on all shifts. The heat from the machines and human activity should make work there within the comfort range. Many factory workers already complain about the warm temperatures in their work spaces.

Our heating gas bill-and probably yours at home-is a major expense. Prices for liquefied gas have risen by more than 30% since last winter. Please help us to reduce costs and keep everyone at Duvall at work.

448 Practical Guide to the NEW TOEIC® Test

Page 97: practical guide to the new toeic_practice tests.pdf

186. What kind of company sent the bill? 189. What is the purpose of the new

(A) Energy company policy?

(B) Transportation (A) To prepare for warmer spring

(C) Entertainment weather

(D) Manufacturing (B) To improve building security (C) To make workers feel more

187. On what basis was the charge of comfortable

$10,459.45 calculated? (D) To reduce energy bill costs ;' (A .-+

(A) The current meter reading 190. What is implied in the last sentence of N

(B) The last meter reading (C) The next scheduled meter reading

the last paragraph of the notice?

(D) Payment due (A) Everyone should be busy at their 1

work. 2

188. At what temperature will office workers (B) Management is working hard to

work in? keep warm. 3

(A) 70oF (C) Reducing the heating bill may save 4

(B) 74oF jobs. 5

(C) 75°F (D) Lazy workers will be fired.

6 (D) 77oF -

Simulated TOEIC Test 2 Reading 449

Page 98: practical guide to the new toeic_practice tests.pdf

Questions 191-1 95 refer to the following document and e-mail message.

Idaho Department of Public Health 74 Eagle Boulevard, Boise, Idaho 83707

Telephone: (208) 544-2138 www.state.id/ dph.us

Advisory on Blue Mountain Mite Infestation

Random tests conducted on potato fields by the Idaho State Agriculture

Department have revealed the presence of the Blue Mountain mite in southern

Idaho. Though short of a warning, we are advising all those living in those areas to be aware of the possible spread of these harmful arachnids.

The Blue Mountain mite inhabits sandy soil. In addition to causing damage to

potato crops, this mite has been known to cause illness in humans. The Blue

Mountain mite is nearly impossible to notice, especially as its light brown skin

blends easily with that of the soil. It is mildly venomous. All those working in or

near farms or handling agriculture products in the above-mentioned areas should

take proper precautions. Where mite infestation is noticed, property owners should contact us by Internet or by our toll-free number: 1-800-544-MITE.

State Agricultural Officials

.-~----------------------------------~ Warner Simmons

Possible Blue Mountain Mite Infestation

Dear Sirs:

I think I may have Blue Mountain mite infestation on several hundred acres of my property

here in Gooding. I'm a potato farmer, and I've noticed what I think are insect bites from

time to time. However, I've never seen the insects. They leave a small hole with red dirt

around it. I'm worried that this could be the Blue Mountain mite, but I've never been sick

from them.

My crops have suffered an average amount of damage, but nothing out of the ordinary.

Do you want me to prepare a few samples of my soil or potatoes, or will you do that

yourselves? Please contact me at home at 421-8001 , or reply by e-mail. I'm awaiting your

response. Thank you very much for your attention.

450 Practical Guide to the NEW TOEIC® Test

Page 99: practical guide to the new toeic_practice tests.pdf

191. What is this advisory mainly about?

(A) An oncoming hurricane (B) Massive crop damage to potatoes (C) A serious traffic accident (D) A possible infestation of a pest

192. Which of the following is true about the Blue Mountain mite?

(A) Its blue color makes it easy to identify.

(B) Its venom is slightly poisonous. (C) It lives high atop mountains. (D) It lives inside potatoes.

193. In the advisory, the word "nearly" in paragraph 2, line 3 is closest in meaning to

(A) barely (B) definitely (C) almost (D) unexpectedly

194. Why does Mr. Simmons think he may have Blue Mountain mite infestation?

(A) His potato crop recently failed. (B) He has been bitten by field pests. (C) His son has suffered pest bites. (D) Government officials tested his land.

195. What does Mr. Simmons offer to do?

(A) Plant a crop other than potatoes (B) Move to another state (C) Invite some government officials to

stay on his farm (D) Provide samples from his farm

Simulated TOEIC Test 2 Reading 451

2

3

4

5

6 -

Page 100: practical guide to the new toeic_practice tests.pdf

Questions 196-200 refer to the following advertisement and memo.

You've been there. You walk into a friend's entertainment center and almost trip over

the wires all over the floor. Hulking around the room are enormous, black speakers. Or

maybe this describes your own living room. It doesn't have to be this way.

Thanks to the Jazz Hipster Corporation, innovations in speakers allow for a more

modern, convenient, and affordable sound solution at every sound enthusiast's fingertips.

Jazz Hipster has produced a wide range of speakers for the plugged-in, on-the-go generation.

Their Mobile Theater was designed for computer users, vastly improving the sound available

while working or playing. Their Sound Walker speaker for notebook computers is so small

that it can fit on your palm but provides better audio quality than regular notebooks can.

For the home stereo set, what could be more appealing than bright and

attractive, space-age design, aluminum-covered speakers? A small remote

control device gives the owner complete control over all the speakers with its

wireless transmission function. Gone are those wire death traps all over the

floor, and in comes a world-class sound system. Don't hesitate to experience

great sound. Visit the showroom today.

Date:

To:

December 5

Paula

From: Henry

MEMO

Subject: Christmas Office Party

I just read a newspaper article on some high-quality, inexpensive

speakers. I was wondering if we have enough money in the budget

to buy a stereo set to play music for the upcoming Christmas party

and other parties in the future. A company called Jazz Hipster makes

great products, many of them using new technology that reduces wire

hazards while providing great sound. Last year, Melinda tripped over a

speaker wire, and many of our colleagues complained about the poor

sound quality.

I think we can use money for this from the employees' union budget

and the company's employees' safety budget. I'm going to take a look

at their showroom this weekend. I'll follow up with you on Monday. In

the meantime, why don't you approach Martha about the possibility of

purchasing a new sound system?

452 Practical Guide to the NEW TOEIC® Test

Page 101: practical guide to the new toeic_practice tests.pdf

196.

197.

198.

According to the advertisement, what is 199. Why did Henry send a memo to Paula? the problem with traditional speakers? (A) To remind her of the office Christmas (A) They are too costly. party (B) They are too bulky. (B) To tell her about his new Jazz (C) They are too loud. Hipster speakers (D) They are too cutting-edge. (C) To suggest buying Jazz Hipster

speakers for a party What innovation has Jazz Hipster made (D) To borrow money to buy Jazz in computer speakers? Hipster speakers

(A) They have reduced their size. How does Henry plan to afford the Jazz (B) They have made them brighter. 200.

(C) They eliminated remote control Hipster speakers?

units. (A) He will use money from the

(D) They require an adapter. Christmas party fund . (B) He will use money from two office

What is one difference between budgets. traditional speakers and those of Jazz (C) He will use money from trading in Hipster's? old speakers.

(A) Jazz Hipster speakers are the size (D) He will borrow money from his

of fingertips. parents.

(B) Jazz Hipster speakers are made for movie theaters.

(C) Traditional speakers produce better

sound. (D) Traditional speakers look dark and

heavy.

Stop! This is the end of the test. If you finish before time is called, you may go back to Parts 5, 6, and 7 and check your work.

I GO ON TO THE NEXT PAGE

Simulated TOEIC Test 2 Reading 453

;t en -N

2

3

4

5

6

Page 102: practical guide to the new toeic_practice tests.pdf

454 Practical Guide to the NEW TOEIC® Test